Как найти касательную к окружности через производную

Определение формулы касательной к окружности

Коэффициенты окружности
Точка на окружности, через которую надо провести касательную
Общая формула окружности
Уравнение касательной в указанной точке

Касательная к окружности

Если не использовать понятие производной, и взять объяснение из учебников середины прошлого века, то “Касательная к окружности – это прямая пересекающая окружность в двух совпадающих точках”

Окружность на плоскости может быть представлена в виде нескольких исходных данных

1. В виде координат центра окружности (x0,y0) и её радиуса R.

2. В виде общего уравнения

В виде параметрического вида и в полярных координатах мы рассматривать не будем, так как там формулы тоже на базируются на координатах центра окружности и радиусе.

Наша задача, зная параметры окружности и точку принадлежащую этой окружности вычислить параметры касательной к этой окружности.

Эта задача, является частным решением более общего калькулятор касательная к кривой второго порядка

Итак, если окружность выражена формулой

Уравнение касательной к окружности если нам известны параметры общего уравнения таково:

Таким образом, зная все коэффициенты, мы очень легко найдем уравнение касательной в заданной точке.

ВАЖНО: При указании точки, она должна быть обязательно(!!) принадлежать окружности,
и не быть точкой в какой либо стороне. В противном случае, уравнение касательной будет неверным.

Примеры

Вычислить уравнение касательной в точке (13.8, 0) к окружности выраженной формулой

Касательная к графику функции в точке. Уравнение касательной. Геометрический смысл производной

Статья дает подробное разъяснение определений, геометрического смысла производной с графическими обозначениями. Будет рассмотрено уравнение касательной прямой с приведением примеров, найдено уравнения касательной к кривым 2 порядка.

Определения и понятия

Угол наклона прямой y = k x + b называется угол α , который отсчитывается от положительного направления оси о х к прямой y = k x + b в положительном направлении.

На рисунке направление о х обозначается при помощи зеленой стрелки и в виде зеленой дуги, а угол наклона при помощи красной дуги. Синяя линия относится к прямой.

Угловой коэффициент прямой y = k x + b называют числовым коэффициентом k .

Угловой коэффициент равняется тангенсу наклона прямой, иначе говоря k = t g α .

  • Угол наклона прямой равняется 0 только при параллельности о х и угловом коэффициенте, равному нулю, потому как тангенс нуля равен 0 . Значит, вид уравнения будет y = b .
  • Если угол наклона прямой y = k x + b острый, тогда выполняются условия 0 α π 2 или 0 ° α 90 ° . Отсюда имеем, что значение углового коэффициента k считается положительным числом, потому как значение тангенс удовлетворяет условию t g α > 0 , причем имеется возрастание графика.
  • Если α = π 2 , тогда расположение прямой перпендикулярно о х . Равенство задается при помощи равенства x = c со значением с , являющимся действительным числом.
  • Если угол наклона прямой y = k x + b тупой, то соответствует условиям π 2 α π или 90 ° α 180 ° , значение углового коэффициента k принимает отрицательное значение, а график убывает.

Определение 3

Секущей называют прямую, которая проходит через 2 точки функции f ( x ) . Иначе говоря, секущая – это прямая, которая проводится через любые две точки графика заданной функции.

По рисунку видно, что А В является секущей, а f ( x ) – черная кривая, α – красная дуга, означающая угол наклона секущей.

Когда угловой коэффициент прямой равняется тангенсу угла наклона, то видно, что тангенс из прямоугольного треугольника А В С можно найти по отношению противолежащего катета к прилежащему.

Получаем формулу для нахождения секущей вида:

k = t g α = B C A C = f ( x B ) – f x A x B – x A , где абсциссами точек А и В являются значения x A , x B , а f ( x A ) , f ( x B ) – это значения функции в этих точках.

Очевидно, что угловой коэффициент секущей определен при помощи равенства k = f ( x B ) – f ( x A ) x B – x A или k = f ( x A ) – f ( x B ) x A – x B , причем уравнение необходимо записать как y = f ( x B ) – f ( x A ) x B – x A · x – x A + f ( x A ) или
y = f ( x A ) – f ( x B ) x A – x B · x – x B + f ( x B ) .

Секущая делит график визуально на 3 части: слева от точки А , от А до В , справа от В . На располагаемом ниже рисунке видно, что имеются три секущие, которые считаются совпадающими, то есть задаются при помощи аналогичного уравнения.

По определению видно, что прямая и ее секущая в данном случае совпадают.

Секущая может множественно раз пересекать график заданной функции. Если имеется уравнение вида у = 0 для секущей, тогда количество точек пересечения с синусоидой бесконечно.

Касательная к графику функции f ( x ) в точке x 0 ; f ( x 0 ) называется прямая, проходящая через заданную точку x 0 ; f ( x 0 ) , с наличием отрезка, который имеет множество значений х , близких к x 0 .

Рассмотрим подробно на ниже приведенном примере. Тогда видно, что прямая, заданная функцией y = x + 1 , считается касательной к y = 2 x в точке с координатами ( 1 ; 2 ) . Для наглядности, необходимо рассмотреть графики с приближенными к ( 1 ; 2 ) значениями. Функция y = 2 x обозначена черным цветом, синяя линия – касательная, красная точка – точка пересечения.

Очевидно, что y = 2 x сливается с прямой у = х + 1 .

Для определения касательной следует рассмотреть поведение касательной А В при бесконечном приближении точки В к точке А . Для наглядности приведем рисунок.

Секущая А В , обозначенная при помощи синей линии, стремится к положению самой касательной, а угол наклона секущей α начнет стремиться к углу наклона самой касательной α x .

Касательной к графику функции y = f ( x ) в точке А считается предельное положение секущей А В при В стремящейся к А , то есть B → A .

Теперь перейдем к рассмотрению геометрического смысла производной функции в точке.

Геометрический смысл производной функции в точке

Перейдем к рассмотрению секущей А В для функции f ( x ) , где А и В с координатами x 0 , f ( x 0 ) и x 0 + ∆ x , f ( x 0 + ∆ x ) , а ∆ x обозначаем как приращение аргумента. Теперь функция примет вид ∆ y = ∆ f ( x ) = f ( x 0 + ∆ x ) – f ( ∆ x ) . Для наглядности приведем в пример рисунок.

Рассмотрим полученный прямоугольный треугольник А В С . Используем определение тангенса для решения, то есть получим отношение ∆ y ∆ x = t g α . Из определения касательной следует, что lim ∆ x → 0 ∆ y ∆ x = t g α x . По правилу производной в точке имеем, что производную f ( x ) в точке x 0 называют пределом отношений приращения функции к приращению аргумента, где ∆ x → 0 , тогда обозначим как f ( x 0 ) = lim ∆ x → 0 ∆ y ∆ x .

Отсюда следует, что f ‘ ( x 0 ) = lim ∆ x → 0 ∆ y ∆ x = t g α x = k x , где k x обозначают в качестве углового коэффициента касательной.

То есть получаем, что f ’ ( x ) может существовать в точке x 0 причем как и касательная к заданному графику функции в точке касания равной x 0 , f 0 ( x 0 ) , где значение углового коэффициента касательной в точке равняется производной в точке x 0 . Тогда получаем, что k x = f ‘ ( x 0 ) .

Геометрический смысл производной функции в точке в том, что дается понятие существования касательной к графику в этой же точке.

Уравнение касательной прямой

Чтобы записать уравнение любой прямой на плоскости, необходимо иметь угловой коэффициент с точкой, через которую она проходит. Его обозначение принимается как x 0 при пересечении.

Уравнение касательной к графику функции y = f ( x ) в точке x 0 , f 0 ( x 0 ) принимает вид y = f ‘ ( x 0 ) · x – x 0 + f ( x 0 ) .

Имеется в виду, что конечным значением производной f ‘ ( x 0 ) можно определить положение касательной, то есть вертикально при условии lim x → x 0 + 0 f ‘ ( x ) = ∞ и lim x → x 0 – 0 f ‘ ( x ) = ∞ или отсутствие вовсе при условии lim x → x 0 + 0 f ‘ ( x ) ≠ lim x → x 0 – 0 f ‘ ( x ) .

Расположение касательной зависит от значения ее углового коэффициента k x = f ‘ ( x 0 ) . При параллельности к оси о х получаем, что k k = 0 , при параллельности к о у – k x = ∞ , причем вид уравнения касательной x = x 0 возрастает при k x > 0 , убывает при k x 0 .

Произвести составление уравнения касательной к графику функции y = e x + 1 + x 3 3 – 6 – 3 3 x – 17 – 3 3 в точке с координатами ( 1 ; 3 ) с определением угла наклона.

Решение

По условию имеем, что функция определяется для всех действительных чисел. Получаем, что точка с координатами, заданными по условию, ( 1 ; 3 ) является точкой касания, тогда x 0 = – 1 , f ( x 0 ) = – 3 .

Необходимо найти производную в точке со значением – 1 . Получаем, что

y ‘ = e x + 1 + x 3 3 – 6 – 3 3 x – 17 – 3 3 ‘ = = e x + 1 ‘ + x 3 3 ‘ – 6 – 3 3 x ‘ – 17 – 3 3 ‘ = e x + 1 + x 2 – 6 – 3 3 y ‘ ( x 0 ) = y ‘ ( – 1 ) = e – 1 + 1 + – 1 2 – 6 – 3 3 = 3 3

Значение f ’ ( x ) в точке касания является угловым коэффициентом касательной, который равняется тангенсу наклона.

Тогда k x = t g α x = y ‘ ( x 0 ) = 3 3

Отсюда следует, что α x = a r c t g 3 3 = π 6

Ответ: уравнение касательной приобретает вид

y = f ‘ ( x 0 ) · x – x 0 + f ( x 0 ) y = 3 3 ( x + 1 ) – 3 y = 3 3 x – 9 – 3 3

Для наглядности приведем пример в графической иллюстрации.

Черный цвет используется для графика исходной функции, синий цвет – изображение касательной, красная точка – точка касания. Рисунок, располагаемый справа, показывает в увеличенном виде.

Выяснить наличие существования касательной к графику заданной функции
y = 3 · x – 1 5 + 1 в точке с координатами ( 1 ; 1 ) . Составить уравнение и определить угол наклона.

Решение

По условию имеем, что областью определения заданной функции считается множество всех действительных чисел.

Перейдем к нахождению производной

y ‘ = 3 · x – 1 5 + 1 ‘ = 3 · 1 5 · ( x – 1 ) 1 5 – 1 = 3 5 · 1 ( x – 1 ) 4 5

Если x 0 = 1 , тогда f ’ ( x ) не определена, но пределы записываются как lim x → 1 + 0 3 5 · 1 ( x – 1 ) 4 5 = 3 5 · 1 ( + 0 ) 4 5 = 3 5 · 1 + 0 = + ∞ и lim x → 1 – 0 3 5 · 1 ( x – 1 ) 4 5 = 3 5 · 1 ( – 0 ) 4 5 = 3 5 · 1 + 0 = + ∞ , что означает существование вертикальной касательной в точке ( 1 ; 1 ) .

Ответ: уравнение примет вид х = 1 , где угол наклона будет равен π 2 .

Для наглядности изобразим графически.

Найти точки графика функции y = 1 15 x + 2 3 – 4 5 x 2 – 16 5 x – 26 5 + 3 x + 2 , где

  1. Касательная не существует;
  2. Касательная располагается параллельно о х ;
  3. Касательная параллельна прямой y = 8 5 x + 4 .

Решение

Необходимо обратить внимание на область определения. По условию имеем, что функция определена на множестве всех действительных чисел. Раскрываем модуль и решаем систему с промежутками x ∈ – ∞ ; 2 и [ – 2 ; + ∞ ) . Получаем, что

y = – 1 15 x 3 + 18 x 2 + 105 x + 176 , x ∈ – ∞ ; – 2 1 15 x 3 – 6 x 2 + 9 x + 12 , x ∈ [ – 2 ; + ∞ )

Необходимо продифференцировать функцию. Имеем, что

y ‘ = – 1 15 x 3 + 18 x 2 + 105 x + 176 ‘ , x ∈ – ∞ ; – 2 1 15 x 3 – 6 x 2 + 9 x + 12 ‘ , x ∈ [ – 2 ; + ∞ ) ⇔ y ‘ = – 1 5 ( x 2 + 12 x + 35 ) , x ∈ – ∞ ; – 2 1 5 x 2 – 4 x + 3 , x ∈ [ – 2 ; + ∞ )

Когда х = – 2 , тогда производная не существует, потому что односторонние пределы не равны в этой точке:

lim x → – 2 – 0 y ‘ ( x ) = lim x → – 2 – 0 – 1 5 ( x 2 + 12 x + 35 = – 1 5 ( – 2 ) 2 + 12 ( – 2 ) + 35 = – 3 lim x → – 2 + 0 y ‘ ( x ) = lim x → – 2 + 0 1 5 ( x 2 – 4 x + 3 ) = 1 5 – 2 2 – 4 – 2 + 3 = 3

Вычисляем значение функции в точке х = – 2 , где получаем, что

  1. y ( – 2 ) = 1 15 – 2 + 2 3 – 4 5 ( – 2 ) 2 – 16 5 ( – 2 ) – 26 5 + 3 – 2 + 2 = – 2 , то есть касательная в точке ( – 2 ; – 2 ) не будет существовать.
  2. Касательная параллельна о х , когда угловой коэффициент равняется нулю. Тогда k x = t g α x = f ‘ ( x 0 ) . То есть необходимо найти значения таких х , когда производная функции обращает ее в ноль. То есть значения f ’ ( x ) и будут являться точками касания, где касательная является параллельной о х .

Когда x ∈ – ∞ ; – 2 , тогда – 1 5 ( x 2 + 12 x + 35 ) = 0 , а при x ∈ ( – 2 ; + ∞ ) получаем 1 5 ( x 2 – 4 x + 3 ) = 0 .

– 1 5 ( x 2 + 12 x + 35 ) = 0 D = 12 2 – 4 · 35 = 144 – 140 = 4 x 1 = – 12 + 4 2 = – 5 ∈ – ∞ ; – 2 x 2 = – 12 – 4 2 = – 7 ∈ – ∞ ; – 2 1 5 ( x 2 – 4 x + 3 ) = 0 D = 4 2 – 4 · 3 = 4 x 3 = 4 – 4 2 = 1 ∈ – 2 ; + ∞ x 4 = 4 + 4 2 = 3 ∈ – 2 ; + ∞

Вычисляем соответствующие значения функции

y 1 = y – 5 = 1 15 – 5 + 2 3 – 4 5 – 5 2 – 16 5 – 5 – 26 5 + 3 – 5 + 2 = 8 5 y 2 = y ( – 7 ) = 1 15 – 7 + 2 3 – 4 5 ( – 7 ) 2 – 16 5 – 7 – 26 5 + 3 – 7 + 2 = 4 3 y 3 = y ( 1 ) = 1 15 1 + 2 3 – 4 5 · 1 2 – 16 5 · 1 – 26 5 + 3 1 + 2 = 8 5 y 4 = y ( 3 ) = 1 15 3 + 2 3 – 4 5 · 3 2 – 16 5 · 3 – 26 5 + 3 3 + 2 = 4 3

Отсюда – 5 ; 8 5 , – 4 ; 4 3 , 1 ; 8 5 , 3 ; 4 3 считаются искомыми точками графика функции.

Рассмотрим графическое изображение решения.

Черная линия – график функции, красные точки – точки касания.

  1. Когда прямые располагаются параллельно, то угловые коэффициенты равны. Тогда необходимо заняться поиском точек графика функции, где угловой коэффициент будет равняться значению 8 5 . Для этого нужно решить уравнение вида y ‘ ( x ) = 8 5 . Тогда, если x ∈ – ∞ ; – 2 , получаем, что – 1 5 ( x 2 + 12 x + 35 ) = 8 5 , а если x ∈ ( – 2 ; + ∞ ) , тогда 1 5 ( x 2 – 4 x + 3 ) = 8 5 .

Первое уравнение не имеет корней, так как дискриминант меньше нуля. Запишем, что

– 1 5 x 2 + 12 x + 35 = 8 5 x 2 + 12 x + 43 = 0 D = 12 2 – 4 · 43 = – 28 0

Другое уравнение имеет два действительных корня, тогда

1 5 ( x 2 – 4 x + 3 ) = 8 5 x 2 – 4 x – 5 = 0 D = 4 2 – 4 · ( – 5 ) = 36 x 1 = 4 – 36 2 = – 1 ∈ – 2 ; + ∞ x 2 = 4 + 36 2 = 5 ∈ – 2 ; + ∞

Перейдем к нахождению значений функции. Получаем, что

y 1 = y ( – 1 ) = 1 15 – 1 + 2 3 – 4 5 ( – 1 ) 2 – 16 5 ( – 1 ) – 26 5 + 3 – 1 + 2 = 4 15 y 2 = y ( 5 ) = 1 15 5 + 2 3 – 4 5 · 5 2 – 16 5 · 5 – 26 5 + 3 5 + 2 = 8 3

Точки со значениями – 1 ; 4 15 , 5 ; 8 3 являются точками, в которых касательные параллельны прямой y = 8 5 x + 4 .

Ответ: черная линия – график функции, красная линия – график y = 8 5 x + 4 , синяя линия – касательные в точках – 1 ; 4 15 , 5 ; 8 3 .

Возможно существование бесконечного количества касательных для заданных функций.

Написать уравнения всех имеющихся касательных функции y = 3 cos 3 2 x – π 4 – 1 3 , которые располагаются перпендикулярно прямой y = – 2 x + 1 2 .

Решение

Для составления уравнения касательной необходимо найти коэффициент и координаты точки касания, исходя из условия перпендикулярности прямых. Определение звучит так: произведение угловых коэффициентов, которые перпендикулярны прямым, равняется – 1 , то есть записывается как k x · k ⊥ = – 1 . Из условия имеем, что угловой коэффициент располагается перпендикулярно прямой и равняется k ⊥ = – 2 , тогда k x = – 1 k ⊥ = – 1 – 2 = 1 2 .

Теперь необходимо найти координаты точек касания. Нужно найти х , после чего его значение для заданной функции. Отметим, что из геометрического смысла производной в точке
x 0 получаем, что k x = y ‘ ( x 0 ) . Из данного равенства найдем значения х для точек касания.

y ‘ ( x 0 ) = 3 cos 3 2 x 0 – π 4 – 1 3 ‘ = 3 · – sin 3 2 x 0 – π 4 · 3 2 x 0 – π 4 ‘ = = – 3 · sin 3 2 x 0 – π 4 · 3 2 = – 9 2 · sin 3 2 x 0 – π 4 ⇒ k x = y ‘ ( x 0 ) ⇔ – 9 2 · sin 3 2 x 0 – π 4 = 1 2 ⇒ sin 3 2 x 0 – π 4 = – 1 9

Это тригонометрическое уравнение будет использовано для вычисления ординат точек касания.

3 2 x 0 – π 4 = a r c sin – 1 9 + 2 πk или 3 2 x 0 – π 4 = π – a r c sin – 1 9 + 2 πk

3 2 x 0 – π 4 = – a r c sin 1 9 + 2 πk или 3 2 x 0 – π 4 = π + a r c sin 1 9 + 2 πk

x 0 = 2 3 π 4 – a r c sin 1 9 + 2 πk или x 0 = 2 3 5 π 4 + a r c sin 1 9 + 2 πk , k ∈ Z

Z – множество целых чисел.

Найдены х точек касания. Теперь необходимо перейти к поиску значений у :

y 0 = 3 cos 3 2 x 0 – π 4 – 1 3

y 0 = 3 · 1 – sin 2 3 2 x 0 – π 4 – 1 3 или y 0 = 3 · – 1 – sin 2 3 2 x 0 – π 4 – 1 3

y 0 = 3 · 1 – – 1 9 2 – 1 3 или y 0 = 3 · – 1 – – 1 9 2 – 1 3

y 0 = 4 5 – 1 3 или y 0 = – 4 5 + 1 3

Отсюда получаем, что 2 3 π 4 – a r c sin 1 9 + 2 πk ; 4 5 – 1 3 , 2 3 5 π 4 + a r c sin 1 9 + 2 πk ; – 4 5 + 1 3 являются точками касания.

Ответ: необходимы уравнения запишутся как

y = 1 2 x – 2 3 π 4 – a r c sin 1 9 + 2 πk + 4 5 – 1 3 , y = 1 2 x – 2 3 5 π 4 + a r c sin 1 9 + 2 πk – 4 5 + 1 3 , k ∈ Z

Для наглядного изображения рассмотрим функцию и касательную на координатной прямой.

Рисунок показывает, что расположение функции идет на промежутке [ – 10 ; 10 ] , где черная прямя – график функции, синие линии – касательные, которые располагаются перпендикулярно заданной прямой вида y = – 2 x + 1 2 . Красные точки – это точки касания.

Касательная к окружности, эллипсу, гиперболе, параболе

Канонические уравнения кривых 2 порядка не являются однозначными функциями. Уравнения касательных для них составляются по известным схемам.

Касательная к окружности

Для задания окружности с центром в точке x c e n t e r ; y c e n t e r и радиусом R применяется формула x – x c e n t e r 2 + y – y c e n t e r 2 = R 2 .

Данное равенство может быть записано как объединение двух функций:

y = R 2 – x – x c e n t e r 2 + y c e n t e r y = – R 2 – x – x c e n t e r 2 + y c e n t e r

Первая функция располагается вверху, а вторая внизу, как показано на рисунке.

Для составления уравнения окружности в точке x 0 ; y 0 , которая располагается в верхней или нижней полуокружности, следует найти уравнение графика функции вида y = R 2 – x – x c e n t e r 2 + y c e n t e r или y = – R 2 – x – x c e n t e r 2 + y c e n t e r в указанной точке.

Когда в точках x c e n t e r ; y c e n t e r + R и x c e n t e r ; y c e n t e r – R касательные могут быть заданы уравнениями y = y c e n t e r + R и y = y c e n t e r – R , а в точках x c e n t e r + R ; y c e n t e r и
x c e n t e r – R ; y c e n t e r будут являться параллельными о у , тогда получим уравнения вида x = x c e n t e r + R и x = x c e n t e r – R .

Касательная к эллипсу

Когда эллипс имеет центр в точке x c e n t e r ; y c e n t e r с полуосями a и b , тогда он может быть задан при помощи уравнения x – x c e n t e r 2 a 2 + y – y c e n t e r 2 b 2 = 1 .

Эллипс и окружность могут быть обозначаться при помощи объединения двух функций, а именно: верхнего и нижнего полуэллипса. Тогда получаем, что

y = b a · a 2 – ( x – x c e n t e r ) 2 + y c e n t e r y = – b a · a 2 – ( x – x c e n t e r ) 2 + y c e n t e r

Если касательные располагаются на вершинах эллипса, тогда они параллельны о х или о у . Ниже для наглядности рассмотрим рисунок.

Написать уравнение касательной к эллипсу x – 3 2 4 + y – 5 2 25 = 1 в точках со значениями x равного х = 2 .

Решение

Необходимо найти точки касания, которые соответствуют значению х = 2 . Производим подстановку в имеющееся уравнение эллипса и получаем, что

x – 3 2 4 x = 2 + y – 5 2 25 = 1 1 4 + y – 5 2 25 = 1 ⇒ y – 5 2 = 3 4 · 25 ⇒ y = ± 5 3 2 + 5

Тогда 2 ; 5 3 2 + 5 и 2 ; – 5 3 2 + 5 являются точками касания, которые принадлежат верхнему и нижнему полуэллипсу.

Перейдем к нахождению и разрешению уравнения эллипса относительно y . Получим, что

x – 3 2 4 + y – 5 2 25 = 1 y – 5 2 25 = 1 – x – 3 2 4 ( y – 5 ) 2 = 25 · 1 – x – 3 2 4 y – 5 = ± 5 · 1 – x – 3 2 4 y = 5 ± 5 2 4 – x – 3 2

Очевидно, что верхний полуэллипс задается с помощью функции вида y = 5 + 5 2 4 – x – 3 2 , а нижний y = 5 – 5 2 4 – x – 3 2 .

Применим стандартный алгоритм для того, чтобы составить уравнение касательной к графику функции в точке. Запишем, что уравнение для первой касательной в точке 2 ; 5 3 2 + 5 будет иметь вид

y ‘ = 5 + 5 2 4 – x – 3 2 ‘ = 5 2 · 1 2 4 – ( x – 3 ) 2 · 4 – ( x – 3 ) 2 ‘ = = – 5 2 · x – 3 4 – ( x – 3 ) 2 ⇒ y ‘ ( x 0 ) = y ‘ ( 2 ) = – 5 2 · 2 – 3 4 – ( 2 – 3 ) 2 = 5 2 3 ⇒ y = y ‘ ( x 0 ) · x – x 0 + y 0 ⇔ y = 5 2 3 ( x – 2 ) + 5 3 2 + 5

Получаем, что уравнение второй касательной со значением в точке
2 ; – 5 3 2 + 5 принимает вид

y ‘ = 5 – 5 2 4 – ( x – 3 ) 2 ‘ = – 5 2 · 1 2 4 – ( x – 3 ) 2 · 4 – ( x – 3 ) 2 ‘ = = 5 2 · x – 3 4 – ( x – 3 ) 2 ⇒ y ‘ ( x 0 ) = y ‘ ( 2 ) = 5 2 · 2 – 3 4 – ( 2 – 3 ) 2 = – 5 2 3 ⇒ y = y ‘ ( x 0 ) · x – x 0 + y 0 ⇔ y = – 5 2 3 ( x – 2 ) – 5 3 2 + 5

Графически касательные обозначаются так:

Касательная к гиперболе

Когда гипербола имеет центр в точке x c e n t e r ; y c e n t e r и вершины x c e n t e r + α ; y c e n t e r и x c e n t e r – α ; y c e n t e r , имеет место задание неравенства x – x c e n t e r 2 α 2 – y – y c e n t e r 2 b 2 = 1 , если с вершинами x c e n t e r ; y c e n t e r + b и x c e n t e r ; y c e n t e r – b , тогда задается при помощи неравенства x – x c e n t e r 2 α 2 – y – y c e n t e r 2 b 2 = – 1 .

Гипербола может быть представлена в виде двух объединенных функций вида

y = b a · ( x – x c e n t e r ) 2 – a 2 + y c e n t e r y = – b a · ( x – x c e n t e r ) 2 – a 2 + y c e n t e r или y = b a · ( x – x c e n t e r ) 2 + a 2 + y c e n t e r y = – b a · ( x – x c e n t e r ) 2 + a 2 + y c e n t e r

В первом случае имеем, что касательные параллельны о у , а во втором параллельны о х .

Отсюда следует, что для того, чтобы найти уравнение касательной к гиперболе, необходимо выяснить, какой функции принадлежит точка касания. Чтобы определить это, необходимо произвести подстановку в уравнения и проверить их на тождественность.

Составить уравнение касательной к гиперболе x – 3 2 4 – y + 3 2 9 = 1 в точке 7 ; – 3 3 – 3 .

Решение

Необходимо преобразовать запись решения нахождения гиперболы при помощи 2 функций. Получим, что

x – 3 2 4 – y + 3 2 9 = 1 ⇒ y + 3 2 9 = x – 3 2 4 – 1 ⇒ y + 3 2 = 9 · x – 3 2 4 – 1 ⇒ y + 3 = 3 2 · x – 3 2 – 4 и л и y + 3 = – 3 2 · x – 3 2 – 4 ⇒ y = 3 2 · x – 3 2 – 4 – 3 y = – 3 2 · x – 3 2 – 4 – 3

Необходимо выявить, к какой функции принадлежит заданная точка с координатами 7 ; – 3 3 – 3 .

Очевидно, что для проверки первой функции необходимо y ( 7 ) = 3 2 · ( 7 – 3 ) 2 – 4 – 3 = 3 3 – 3 ≠ – 3 3 – 3 , тогда точка графику не принадлежит, так как равенство не выполняется.

Для второй функции имеем, что y ( 7 ) = – 3 2 · ( 7 – 3 ) 2 – 4 – 3 = – 3 3 – 3 ≠ – 3 3 – 3 , значит, точка принадлежит заданному графику. Отсюда следует найти угловой коэффициент.

y ‘ = – 3 2 · ( x – 3 ) 2 – 4 – 3 ‘ = – 3 2 · x – 3 ( x – 3 ) 2 – 4 ⇒ k x = y ‘ ( x 0 ) = – 3 2 · x 0 – 3 x 0 – 3 2 – 4 x 0 = 7 = – 3 2 · 7 – 3 7 – 3 2 – 4 = – 3

Ответ: уравнение касательной можно представить как

y = – 3 · x – 7 – 3 3 – 3 = – 3 · x + 4 3 – 3

Наглядно изображается так:

Касательная к параболе

Чтобы составить уравнение касательной к параболе y = a x 2 + b x + c в точке x 0 , y ( x 0 ) , необходимо использовать стандартный алгоритм, тогда уравнение примет вид y = y ‘ ( x 0 ) · x – x 0 + y ( x 0 ) . Такая касательная в вершине параллельна о х .

Следует задать параболу x = a y 2 + b y + c как объединение двух функций. Поэтому нужно разрешить уравнение относительно у . Получаем, что

x = a y 2 + b y + c ⇔ a y 2 + b y + c – x = 0 D = b 2 – 4 a ( c – x ) y = – b + b 2 – 4 a ( c – x ) 2 a y = – b – b 2 – 4 a ( c – x ) 2 a

Графически изобразим как:

Для выяснения принадлежности точки x 0 , y ( x 0 ) функции, нежно действовать по стандартному алгоритму. Такая касательная будет параллельна о у относительно параболы.

Написать уравнение касательной к графику x – 2 y 2 – 5 y + 3 , когда имеем угол наклона касательной 150 ° .

Решение

Начинаем решение с представления параболы в качестве двух функций. Получим, что

– 2 y 2 – 5 y + 3 – x = 0 D = ( – 5 ) 2 – 4 · ( – 2 ) · ( 3 – x ) = 49 – 8 x y = 5 + 49 – 8 x – 4 y = 5 – 49 – 8 x – 4

Значение углового коэффициента равняется значению производной в точке x 0 этой функции и равняется тангенсу угла наклона.

k x = y ‘ ( x 0 ) = t g α x = t g 150 ° = – 1 3

Отсюда определим значение х для точек касания.

Первая функция запишется как

y ‘ = 5 + 49 – 8 x – 4 ‘ = 1 49 – 8 x ⇒ y ‘ ( x 0 ) = 1 49 – 8 x 0 = – 1 3 ⇔ 49 – 8 x 0 = – 3

Очевидно, что действительных корней нет, так как получили отрицательное значение. Делаем вывод, что касательной с углом 150 ° для такой функции не существует.

Вторая функция запишется как

y ‘ = 5 – 49 – 8 x – 4 ‘ = – 1 49 – 8 x ⇒ y ‘ ( x 0 ) = – 1 49 – 8 x 0 = – 1 3 ⇔ 49 – 8 x 0 = – 3 x 0 = 23 4 ⇒ y ( x 0 ) = 5 – 49 – 8 · 23 4 – 4 = – 5 + 3 4

Имеем, что точки касания – 23 4 ; – 5 + 3 4 .

Ответ: уравнение касательной принимает вид

Нахождение касательной к окружности через производную

Определение формулы касательной к окружности

Коэффициенты окружности
Точка на окружности, через которую надо провести касательную
Общая формула окружности
Уравнение касательной в указанной точке

Касательная к окружности

Если не использовать понятие производной, и взять объяснение из учебников середины прошлого века, то «Касательная к окружности — это прямая пересекающая окружность в двух совпадающих точках»

Окружность на плоскости может быть представлена в виде нескольких исходных данных

1. В виде координат центра окружности (x0,y0) и её радиуса R.

2. В виде общего уравнения

В виде параметрического вида и в полярных координатах мы рассматривать не будем, так как там формулы тоже на базируются на координатах центра окружности и радиусе.

Наша задача, зная параметры окружности и точку принадлежащую этой окружности вычислить параметры касательной к этой окружности.

Эта задача, является частным решением более общего калькулятор касательная к кривой второго порядка

Итак, если окружность выражена формулой

Уравнение касательной к окружности если нам известны параметры общего уравнения таково:

Таким образом, зная все коэффициенты, мы очень легко найдем уравнение касательной в заданной точке.

ВАЖНО: При указании точки, она должна быть обязательно(!!) принадлежать окружности,
и не быть точкой в какой либо стороне. В противном случае, уравнение касательной будет неверным.

Примеры

Вычислить уравнение касательной в точке (13.8, 0) к окружности выраженной формулой

Касательная к графику функции в точке. Уравнение касательной. Геометрический смысл производной

Статья дает подробное разъяснение определений, геометрического смысла производной с графическими обозначениями. Будет рассмотрено уравнение касательной прямой с приведением примеров, найдено уравнения касательной к кривым 2 порядка.

Определения и понятия

Угол наклона прямой y = k x + b называется угол α , который отсчитывается от положительного направления оси о х к прямой y = k x + b в положительном направлении.

На рисунке направление о х обозначается при помощи зеленой стрелки и в виде зеленой дуги, а угол наклона при помощи красной дуги. Синяя линия относится к прямой.

Угловой коэффициент прямой y = k x + b называют числовым коэффициентом k .

Угловой коэффициент равняется тангенсу наклона прямой, иначе говоря k = t g α .

  • Угол наклона прямой равняется 0 только при параллельности о х и угловом коэффициенте, равному нулю, потому как тангенс нуля равен 0 . Значит, вид уравнения будет y = b .
  • Если угол наклона прямой y = k x + b острый, тогда выполняются условия 0 α π 2 или 0 ° α 90 ° . Отсюда имеем, что значение углового коэффициента k считается положительным числом, потому как значение тангенс удовлетворяет условию t g α > 0 , причем имеется возрастание графика.
  • Если α = π 2 , тогда расположение прямой перпендикулярно о х . Равенство задается при помощи равенства x = c со значением с , являющимся действительным числом.
  • Если угол наклона прямой y = k x + b тупой, то соответствует условиям π 2 α π или 90 ° α 180 ° , значение углового коэффициента k принимает отрицательное значение, а график убывает.

Секущей называют прямую, которая проходит через 2 точки функции f ( x ) . Иначе говоря, секущая – это прямая, которая проводится через любые две точки графика заданной функции.

По рисунку видно, что А В является секущей, а f ( x ) – черная кривая, α — красная дуга, означающая угол наклона секущей.

Когда угловой коэффициент прямой равняется тангенсу угла наклона, то видно, что тангенс из прямоугольного треугольника А В С можно найти по отношению противолежащего катета к прилежащему.

Получаем формулу для нахождения секущей вида:

k = t g α = B C A C = f ( x B ) — f x A x B — x A , где абсциссами точек А и В являются значения x A , x B , а f ( x A ) , f ( x B ) — это значения функции в этих точках.

Очевидно, что угловой коэффициент секущей определен при помощи равенства k = f ( x B ) — f ( x A ) x B — x A или k = f ( x A ) — f ( x B ) x A — x B , причем уравнение необходимо записать как y = f ( x B ) — f ( x A ) x B — x A · x — x A + f ( x A ) или
y = f ( x A ) — f ( x B ) x A — x B · x — x B + f ( x B ) .

Секущая делит график визуально на 3 части: слева от точки А , от А до В , справа от В . На располагаемом ниже рисунке видно, что имеются три секущие, которые считаются совпадающими, то есть задаются при помощи аналогичного уравнения.

По определению видно, что прямая и ее секущая в данном случае совпадают.

Секущая может множественно раз пересекать график заданной функции. Если имеется уравнение вида у = 0 для секущей, тогда количество точек пересечения с синусоидой бесконечно.

Касательная к графику функции f ( x ) в точке x 0 ; f ( x 0 ) называется прямая, проходящая через заданную точку x 0 ; f ( x 0 ) , с наличием отрезка, который имеет множество значений х , близких к x 0 .

Рассмотрим подробно на ниже приведенном примере. Тогда видно, что прямая, заданная функцией y = x + 1 , считается касательной к y = 2 x в точке с координатами ( 1 ; 2 ) . Для наглядности, необходимо рассмотреть графики с приближенными к ( 1 ; 2 ) значениями. Функция y = 2 x обозначена черным цветом, синяя линия – касательная, красная точка – точка пересечения.

Очевидно, что y = 2 x сливается с прямой у = х + 1 .

Для определения касательной следует рассмотреть поведение касательной А В при бесконечном приближении точки В к точке А . Для наглядности приведем рисунок.

Секущая А В , обозначенная при помощи синей линии, стремится к положению самой касательной, а угол наклона секущей α начнет стремиться к углу наклона самой касательной α x .

Касательной к графику функции y = f ( x ) в точке А считается предельное положение секущей А В при В стремящейся к А , то есть B → A .

Теперь перейдем к рассмотрению геометрического смысла производной функции в точке.

Геометрический смысл производной функции в точке

Перейдем к рассмотрению секущей А В для функции f ( x ) , где А и В с координатами x 0 , f ( x 0 ) и x 0 + ∆ x , f ( x 0 + ∆ x ) , а ∆ x обозначаем как приращение аргумента. Теперь функция примет вид ∆ y = ∆ f ( x ) = f ( x 0 + ∆ x ) — f ( ∆ x ) . Для наглядности приведем в пример рисунок.

Рассмотрим полученный прямоугольный треугольник А В С . Используем определение тангенса для решения, то есть получим отношение ∆ y ∆ x = t g α . Из определения касательной следует, что lim ∆ x → 0 ∆ y ∆ x = t g α x . По правилу производной в точке имеем, что производную f ( x ) в точке x 0 называют пределом отношений приращения функции к приращению аргумента, где ∆ x → 0 , тогда обозначим как f ( x 0 ) = lim ∆ x → 0 ∆ y ∆ x .

Отсюда следует, что f ‘ ( x 0 ) = lim ∆ x → 0 ∆ y ∆ x = t g α x = k x , где k x обозначают в качестве углового коэффициента касательной.

То есть получаем, что f ’ ( x ) может существовать в точке x 0 причем как и касательная к заданному графику функции в точке касания равной x 0 , f 0 ( x 0 ) , где значение углового коэффициента касательной в точке равняется производной в точке x 0 . Тогда получаем, что k x = f ‘ ( x 0 ) .

Геометрический смысл производной функции в точке в том, что дается понятие существования касательной к графику в этой же точке.

Уравнение касательной прямой

Чтобы записать уравнение любой прямой на плоскости, необходимо иметь угловой коэффициент с точкой, через которую она проходит. Его обозначение принимается как x 0 при пересечении.

Уравнение касательной к графику функции y = f ( x ) в точке x 0 , f 0 ( x 0 ) принимает вид y = f ‘ ( x 0 ) · x — x 0 + f ( x 0 ) .

Имеется в виду, что конечным значением производной f ‘ ( x 0 ) можно определить положение касательной, то есть вертикально при условии lim x → x 0 + 0 f ‘ ( x ) = ∞ и lim x → x 0 — 0 f ‘ ( x ) = ∞ или отсутствие вовсе при условии lim x → x 0 + 0 f ‘ ( x ) ≠ lim x → x 0 — 0 f ‘ ( x ) .

Расположение касательной зависит от значения ее углового коэффициента k x = f ‘ ( x 0 ) . При параллельности к оси о х получаем, что k k = 0 , при параллельности к о у — k x = ∞ , причем вид уравнения касательной x = x 0 возрастает при k x > 0 , убывает при k x 0 .

Произвести составление уравнения касательной к графику функции y = e x + 1 + x 3 3 — 6 — 3 3 x — 17 — 3 3 в точке с координатами ( 1 ; 3 ) с определением угла наклона.

Решение

По условию имеем, что функция определяется для всех действительных чисел. Получаем, что точка с координатами, заданными по условию, ( 1 ; 3 ) является точкой касания, тогда x 0 = — 1 , f ( x 0 ) = — 3 .

Необходимо найти производную в точке со значением — 1 . Получаем, что

y ‘ = e x + 1 + x 3 3 — 6 — 3 3 x — 17 — 3 3 ‘ = = e x + 1 ‘ + x 3 3 ‘ — 6 — 3 3 x ‘ — 17 — 3 3 ‘ = e x + 1 + x 2 — 6 — 3 3 y ‘ ( x 0 ) = y ‘ ( — 1 ) = e — 1 + 1 + — 1 2 — 6 — 3 3 = 3 3

Значение f ’ ( x ) в точке касания является угловым коэффициентом касательной, который равняется тангенсу наклона.

Тогда k x = t g α x = y ‘ ( x 0 ) = 3 3

Отсюда следует, что α x = a r c t g 3 3 = π 6

Ответ: уравнение касательной приобретает вид

y = f ‘ ( x 0 ) · x — x 0 + f ( x 0 ) y = 3 3 ( x + 1 ) — 3 y = 3 3 x — 9 — 3 3

Для наглядности приведем пример в графической иллюстрации.

Черный цвет используется для графика исходной функции, синий цвет – изображение касательной, красная точка – точка касания. Рисунок, располагаемый справа, показывает в увеличенном виде.

Выяснить наличие существования касательной к графику заданной функции
y = 3 · x — 1 5 + 1 в точке с координатами ( 1 ; 1 ) . Составить уравнение и определить угол наклона.

Решение

По условию имеем, что областью определения заданной функции считается множество всех действительных чисел.

Перейдем к нахождению производной

y ‘ = 3 · x — 1 5 + 1 ‘ = 3 · 1 5 · ( x — 1 ) 1 5 — 1 = 3 5 · 1 ( x — 1 ) 4 5

Если x 0 = 1 , тогда f ’ ( x ) не определена, но пределы записываются как lim x → 1 + 0 3 5 · 1 ( x — 1 ) 4 5 = 3 5 · 1 ( + 0 ) 4 5 = 3 5 · 1 + 0 = + ∞ и lim x → 1 — 0 3 5 · 1 ( x — 1 ) 4 5 = 3 5 · 1 ( — 0 ) 4 5 = 3 5 · 1 + 0 = + ∞ , что означает существование вертикальной касательной в точке ( 1 ; 1 ) .

Ответ: уравнение примет вид х = 1 , где угол наклона будет равен π 2 .

Для наглядности изобразим графически.

Найти точки графика функции y = 1 15 x + 2 3 — 4 5 x 2 — 16 5 x — 26 5 + 3 x + 2 , где

  1. Касательная не существует;
  2. Касательная располагается параллельно о х ;
  3. Касательная параллельна прямой y = 8 5 x + 4 .

Решение

Необходимо обратить внимание на область определения. По условию имеем, что функция определена на множестве всех действительных чисел. Раскрываем модуль и решаем систему с промежутками x ∈ — ∞ ; 2 и [ — 2 ; + ∞ ) . Получаем, что

y = — 1 15 x 3 + 18 x 2 + 105 x + 176 , x ∈ — ∞ ; — 2 1 15 x 3 — 6 x 2 + 9 x + 12 , x ∈ [ — 2 ; + ∞ )

Необходимо продифференцировать функцию. Имеем, что

y ‘ = — 1 15 x 3 + 18 x 2 + 105 x + 176 ‘ , x ∈ — ∞ ; — 2 1 15 x 3 — 6 x 2 + 9 x + 12 ‘ , x ∈ [ — 2 ; + ∞ ) ⇔ y ‘ = — 1 5 ( x 2 + 12 x + 35 ) , x ∈ — ∞ ; — 2 1 5 x 2 — 4 x + 3 , x ∈ [ — 2 ; + ∞ )

Когда х = — 2 , тогда производная не существует, потому что односторонние пределы не равны в этой точке:

lim x → — 2 — 0 y ‘ ( x ) = lim x → — 2 — 0 — 1 5 ( x 2 + 12 x + 35 = — 1 5 ( — 2 ) 2 + 12 ( — 2 ) + 35 = — 3 lim x → — 2 + 0 y ‘ ( x ) = lim x → — 2 + 0 1 5 ( x 2 — 4 x + 3 ) = 1 5 — 2 2 — 4 — 2 + 3 = 3

Вычисляем значение функции в точке х = — 2 , где получаем, что

  1. y ( — 2 ) = 1 15 — 2 + 2 3 — 4 5 ( — 2 ) 2 — 16 5 ( — 2 ) — 26 5 + 3 — 2 + 2 = — 2 , то есть касательная в точке ( — 2 ; — 2 ) не будет существовать.
  2. Касательная параллельна о х , когда угловой коэффициент равняется нулю. Тогда k x = t g α x = f ‘ ( x 0 ) . То есть необходимо найти значения таких х , когда производная функции обращает ее в ноль. То есть значения f ’ ( x ) и будут являться точками касания, где касательная является параллельной о х .

Когда x ∈ — ∞ ; — 2 , тогда — 1 5 ( x 2 + 12 x + 35 ) = 0 , а при x ∈ ( — 2 ; + ∞ ) получаем 1 5 ( x 2 — 4 x + 3 ) = 0 .

— 1 5 ( x 2 + 12 x + 35 ) = 0 D = 12 2 — 4 · 35 = 144 — 140 = 4 x 1 = — 12 + 4 2 = — 5 ∈ — ∞ ; — 2 x 2 = — 12 — 4 2 = — 7 ∈ — ∞ ; — 2 1 5 ( x 2 — 4 x + 3 ) = 0 D = 4 2 — 4 · 3 = 4 x 3 = 4 — 4 2 = 1 ∈ — 2 ; + ∞ x 4 = 4 + 4 2 = 3 ∈ — 2 ; + ∞

Вычисляем соответствующие значения функции

y 1 = y — 5 = 1 15 — 5 + 2 3 — 4 5 — 5 2 — 16 5 — 5 — 26 5 + 3 — 5 + 2 = 8 5 y 2 = y ( — 7 ) = 1 15 — 7 + 2 3 — 4 5 ( — 7 ) 2 — 16 5 — 7 — 26 5 + 3 — 7 + 2 = 4 3 y 3 = y ( 1 ) = 1 15 1 + 2 3 — 4 5 · 1 2 — 16 5 · 1 — 26 5 + 3 1 + 2 = 8 5 y 4 = y ( 3 ) = 1 15 3 + 2 3 — 4 5 · 3 2 — 16 5 · 3 — 26 5 + 3 3 + 2 = 4 3

Отсюда — 5 ; 8 5 , — 4 ; 4 3 , 1 ; 8 5 , 3 ; 4 3 считаются искомыми точками графика функции.

Рассмотрим графическое изображение решения.

Черная линия – график функции, красные точки – точки касания.

  1. Когда прямые располагаются параллельно, то угловые коэффициенты равны. Тогда необходимо заняться поиском точек графика функции, где угловой коэффициент будет равняться значению 8 5 . Для этого нужно решить уравнение вида y ‘ ( x ) = 8 5 . Тогда, если x ∈ — ∞ ; — 2 , получаем, что — 1 5 ( x 2 + 12 x + 35 ) = 8 5 , а если x ∈ ( — 2 ; + ∞ ) , тогда 1 5 ( x 2 — 4 x + 3 ) = 8 5 .

Первое уравнение не имеет корней, так как дискриминант меньше нуля. Запишем, что

— 1 5 x 2 + 12 x + 35 = 8 5 x 2 + 12 x + 43 = 0 D = 12 2 — 4 · 43 = — 28 0

Другое уравнение имеет два действительных корня, тогда

1 5 ( x 2 — 4 x + 3 ) = 8 5 x 2 — 4 x — 5 = 0 D = 4 2 — 4 · ( — 5 ) = 36 x 1 = 4 — 36 2 = — 1 ∈ — 2 ; + ∞ x 2 = 4 + 36 2 = 5 ∈ — 2 ; + ∞

Перейдем к нахождению значений функции. Получаем, что

y 1 = y ( — 1 ) = 1 15 — 1 + 2 3 — 4 5 ( — 1 ) 2 — 16 5 ( — 1 ) — 26 5 + 3 — 1 + 2 = 4 15 y 2 = y ( 5 ) = 1 15 5 + 2 3 — 4 5 · 5 2 — 16 5 · 5 — 26 5 + 3 5 + 2 = 8 3

Точки со значениями — 1 ; 4 15 , 5 ; 8 3 являются точками, в которых касательные параллельны прямой y = 8 5 x + 4 .

Ответ: черная линия – график функции, красная линия – график y = 8 5 x + 4 , синяя линия – касательные в точках — 1 ; 4 15 , 5 ; 8 3 .

Возможно существование бесконечного количества касательных для заданных функций.

Написать уравнения всех имеющихся касательных функции y = 3 cos 3 2 x — π 4 — 1 3 , которые располагаются перпендикулярно прямой y = — 2 x + 1 2 .

Решение

Для составления уравнения касательной необходимо найти коэффициент и координаты точки касания, исходя из условия перпендикулярности прямых. Определение звучит так: произведение угловых коэффициентов, которые перпендикулярны прямым, равняется — 1 , то есть записывается как k x · k ⊥ = — 1 . Из условия имеем, что угловой коэффициент располагается перпендикулярно прямой и равняется k ⊥ = — 2 , тогда k x = — 1 k ⊥ = — 1 — 2 = 1 2 .

Теперь необходимо найти координаты точек касания. Нужно найти х , после чего его значение для заданной функции. Отметим, что из геометрического смысла производной в точке
x 0 получаем, что k x = y ‘ ( x 0 ) . Из данного равенства найдем значения х для точек касания.

y ‘ ( x 0 ) = 3 cos 3 2 x 0 — π 4 — 1 3 ‘ = 3 · — sin 3 2 x 0 — π 4 · 3 2 x 0 — π 4 ‘ = = — 3 · sin 3 2 x 0 — π 4 · 3 2 = — 9 2 · sin 3 2 x 0 — π 4 ⇒ k x = y ‘ ( x 0 ) ⇔ — 9 2 · sin 3 2 x 0 — π 4 = 1 2 ⇒ sin 3 2 x 0 — π 4 = — 1 9

Это тригонометрическое уравнение будет использовано для вычисления ординат точек касания.

3 2 x 0 — π 4 = a r c sin — 1 9 + 2 πk или 3 2 x 0 — π 4 = π — a r c sin — 1 9 + 2 πk

3 2 x 0 — π 4 = — a r c sin 1 9 + 2 πk или 3 2 x 0 — π 4 = π + a r c sin 1 9 + 2 πk

x 0 = 2 3 π 4 — a r c sin 1 9 + 2 πk или x 0 = 2 3 5 π 4 + a r c sin 1 9 + 2 πk , k ∈ Z

Z — множество целых чисел.

Найдены х точек касания. Теперь необходимо перейти к поиску значений у :

y 0 = 3 cos 3 2 x 0 — π 4 — 1 3

y 0 = 3 · 1 — sin 2 3 2 x 0 — π 4 — 1 3 или y 0 = 3 · — 1 — sin 2 3 2 x 0 — π 4 — 1 3

y 0 = 3 · 1 — — 1 9 2 — 1 3 или y 0 = 3 · — 1 — — 1 9 2 — 1 3

y 0 = 4 5 — 1 3 или y 0 = — 4 5 + 1 3

Отсюда получаем, что 2 3 π 4 — a r c sin 1 9 + 2 πk ; 4 5 — 1 3 , 2 3 5 π 4 + a r c sin 1 9 + 2 πk ; — 4 5 + 1 3 являются точками касания.

Ответ: необходимы уравнения запишутся как

y = 1 2 x — 2 3 π 4 — a r c sin 1 9 + 2 πk + 4 5 — 1 3 , y = 1 2 x — 2 3 5 π 4 + a r c sin 1 9 + 2 πk — 4 5 + 1 3 , k ∈ Z

Для наглядного изображения рассмотрим функцию и касательную на координатной прямой.

Рисунок показывает, что расположение функции идет на промежутке [ — 10 ; 10 ] , где черная прямя – график функции, синие линии – касательные, которые располагаются перпендикулярно заданной прямой вида y = — 2 x + 1 2 . Красные точки – это точки касания.

Касательная к окружности, эллипсу, гиперболе, параболе

Канонические уравнения кривых 2 порядка не являются однозначными функциями. Уравнения касательных для них составляются по известным схемам.

Касательная к окружности

Для задания окружности с центром в точке x c e n t e r ; y c e n t e r и радиусом R применяется формула x — x c e n t e r 2 + y — y c e n t e r 2 = R 2 .

Данное равенство может быть записано как объединение двух функций:

y = R 2 — x — x c e n t e r 2 + y c e n t e r y = — R 2 — x — x c e n t e r 2 + y c e n t e r

Первая функция располагается вверху, а вторая внизу, как показано на рисунке.

Для составления уравнения окружности в точке x 0 ; y 0 , которая располагается в верхней или нижней полуокружности, следует найти уравнение графика функции вида y = R 2 — x — x c e n t e r 2 + y c e n t e r или y = — R 2 — x — x c e n t e r 2 + y c e n t e r в указанной точке.

Когда в точках x c e n t e r ; y c e n t e r + R и x c e n t e r ; y c e n t e r — R касательные могут быть заданы уравнениями y = y c e n t e r + R и y = y c e n t e r — R , а в точках x c e n t e r + R ; y c e n t e r и
x c e n t e r — R ; y c e n t e r будут являться параллельными о у , тогда получим уравнения вида x = x c e n t e r + R и x = x c e n t e r — R .

Касательная к эллипсу

Когда эллипс имеет центр в точке x c e n t e r ; y c e n t e r с полуосями a и b , тогда он может быть задан при помощи уравнения x — x c e n t e r 2 a 2 + y — y c e n t e r 2 b 2 = 1 .

Эллипс и окружность могут быть обозначаться при помощи объединения двух функций, а именно: верхнего и нижнего полуэллипса. Тогда получаем, что

y = b a · a 2 — ( x — x c e n t e r ) 2 + y c e n t e r y = — b a · a 2 — ( x — x c e n t e r ) 2 + y c e n t e r

Если касательные располагаются на вершинах эллипса, тогда они параллельны о х или о у . Ниже для наглядности рассмотрим рисунок.

Написать уравнение касательной к эллипсу x — 3 2 4 + y — 5 2 25 = 1 в точках со значениями x равного х = 2 .

Решение

Необходимо найти точки касания, которые соответствуют значению х = 2 . Производим подстановку в имеющееся уравнение эллипса и получаем, что

x — 3 2 4 x = 2 + y — 5 2 25 = 1 1 4 + y — 5 2 25 = 1 ⇒ y — 5 2 = 3 4 · 25 ⇒ y = ± 5 3 2 + 5

Тогда 2 ; 5 3 2 + 5 и 2 ; — 5 3 2 + 5 являются точками касания, которые принадлежат верхнему и нижнему полуэллипсу.

Перейдем к нахождению и разрешению уравнения эллипса относительно y . Получим, что

x — 3 2 4 + y — 5 2 25 = 1 y — 5 2 25 = 1 — x — 3 2 4 ( y — 5 ) 2 = 25 · 1 — x — 3 2 4 y — 5 = ± 5 · 1 — x — 3 2 4 y = 5 ± 5 2 4 — x — 3 2

Очевидно, что верхний полуэллипс задается с помощью функции вида y = 5 + 5 2 4 — x — 3 2 , а нижний y = 5 — 5 2 4 — x — 3 2 .

Применим стандартный алгоритм для того, чтобы составить уравнение касательной к графику функции в точке. Запишем, что уравнение для первой касательной в точке 2 ; 5 3 2 + 5 будет иметь вид

y ‘ = 5 + 5 2 4 — x — 3 2 ‘ = 5 2 · 1 2 4 — ( x — 3 ) 2 · 4 — ( x — 3 ) 2 ‘ = = — 5 2 · x — 3 4 — ( x — 3 ) 2 ⇒ y ‘ ( x 0 ) = y ‘ ( 2 ) = — 5 2 · 2 — 3 4 — ( 2 — 3 ) 2 = 5 2 3 ⇒ y = y ‘ ( x 0 ) · x — x 0 + y 0 ⇔ y = 5 2 3 ( x — 2 ) + 5 3 2 + 5

Получаем, что уравнение второй касательной со значением в точке
2 ; — 5 3 2 + 5 принимает вид

y ‘ = 5 — 5 2 4 — ( x — 3 ) 2 ‘ = — 5 2 · 1 2 4 — ( x — 3 ) 2 · 4 — ( x — 3 ) 2 ‘ = = 5 2 · x — 3 4 — ( x — 3 ) 2 ⇒ y ‘ ( x 0 ) = y ‘ ( 2 ) = 5 2 · 2 — 3 4 — ( 2 — 3 ) 2 = — 5 2 3 ⇒ y = y ‘ ( x 0 ) · x — x 0 + y 0 ⇔ y = — 5 2 3 ( x — 2 ) — 5 3 2 + 5

Графически касательные обозначаются так:

Касательная к гиперболе

Когда гипербола имеет центр в точке x c e n t e r ; y c e n t e r и вершины x c e n t e r + α ; y c e n t e r и x c e n t e r — α ; y c e n t e r , имеет место задание неравенства x — x c e n t e r 2 α 2 — y — y c e n t e r 2 b 2 = 1 , если с вершинами x c e n t e r ; y c e n t e r + b и x c e n t e r ; y c e n t e r — b , тогда задается при помощи неравенства x — x c e n t e r 2 α 2 — y — y c e n t e r 2 b 2 = — 1 .

Гипербола может быть представлена в виде двух объединенных функций вида

y = b a · ( x — x c e n t e r ) 2 — a 2 + y c e n t e r y = — b a · ( x — x c e n t e r ) 2 — a 2 + y c e n t e r или y = b a · ( x — x c e n t e r ) 2 + a 2 + y c e n t e r y = — b a · ( x — x c e n t e r ) 2 + a 2 + y c e n t e r

В первом случае имеем, что касательные параллельны о у , а во втором параллельны о х .

Отсюда следует, что для того, чтобы найти уравнение касательной к гиперболе, необходимо выяснить, какой функции принадлежит точка касания. Чтобы определить это, необходимо произвести подстановку в уравнения и проверить их на тождественность.

Составить уравнение касательной к гиперболе x — 3 2 4 — y + 3 2 9 = 1 в точке 7 ; — 3 3 — 3 .

Решение

Необходимо преобразовать запись решения нахождения гиперболы при помощи 2 функций. Получим, что

x — 3 2 4 — y + 3 2 9 = 1 ⇒ y + 3 2 9 = x — 3 2 4 — 1 ⇒ y + 3 2 = 9 · x — 3 2 4 — 1 ⇒ y + 3 = 3 2 · x — 3 2 — 4 и л и y + 3 = — 3 2 · x — 3 2 — 4 ⇒ y = 3 2 · x — 3 2 — 4 — 3 y = — 3 2 · x — 3 2 — 4 — 3

Необходимо выявить, к какой функции принадлежит заданная точка с координатами 7 ; — 3 3 — 3 .

Очевидно, что для проверки первой функции необходимо y ( 7 ) = 3 2 · ( 7 — 3 ) 2 — 4 — 3 = 3 3 — 3 ≠ — 3 3 — 3 , тогда точка графику не принадлежит, так как равенство не выполняется.

Для второй функции имеем, что y ( 7 ) = — 3 2 · ( 7 — 3 ) 2 — 4 — 3 = — 3 3 — 3 ≠ — 3 3 — 3 , значит, точка принадлежит заданному графику. Отсюда следует найти угловой коэффициент.

y ‘ = — 3 2 · ( x — 3 ) 2 — 4 — 3 ‘ = — 3 2 · x — 3 ( x — 3 ) 2 — 4 ⇒ k x = y ‘ ( x 0 ) = — 3 2 · x 0 — 3 x 0 — 3 2 — 4 x 0 = 7 = — 3 2 · 7 — 3 7 — 3 2 — 4 = — 3

Ответ: уравнение касательной можно представить как

y = — 3 · x — 7 — 3 3 — 3 = — 3 · x + 4 3 — 3

Наглядно изображается так:

Касательная к параболе

Чтобы составить уравнение касательной к параболе y = a x 2 + b x + c в точке x 0 , y ( x 0 ) , необходимо использовать стандартный алгоритм, тогда уравнение примет вид y = y ‘ ( x 0 ) · x — x 0 + y ( x 0 ) . Такая касательная в вершине параллельна о х .

Следует задать параболу x = a y 2 + b y + c как объединение двух функций. Поэтому нужно разрешить уравнение относительно у . Получаем, что

x = a y 2 + b y + c ⇔ a y 2 + b y + c — x = 0 D = b 2 — 4 a ( c — x ) y = — b + b 2 — 4 a ( c — x ) 2 a y = — b — b 2 — 4 a ( c — x ) 2 a

Графически изобразим как:

Для выяснения принадлежности точки x 0 , y ( x 0 ) функции, нежно действовать по стандартному алгоритму. Такая касательная будет параллельна о у относительно параболы.

Написать уравнение касательной к графику x — 2 y 2 — 5 y + 3 , когда имеем угол наклона касательной 150 ° .

Решение

Начинаем решение с представления параболы в качестве двух функций. Получим, что

— 2 y 2 — 5 y + 3 — x = 0 D = ( — 5 ) 2 — 4 · ( — 2 ) · ( 3 — x ) = 49 — 8 x y = 5 + 49 — 8 x — 4 y = 5 — 49 — 8 x — 4

Значение углового коэффициента равняется значению производной в точке x 0 этой функции и равняется тангенсу угла наклона.

k x = y ‘ ( x 0 ) = t g α x = t g 150 ° = — 1 3

Отсюда определим значение х для точек касания.

Первая функция запишется как

y ‘ = 5 + 49 — 8 x — 4 ‘ = 1 49 — 8 x ⇒ y ‘ ( x 0 ) = 1 49 — 8 x 0 = — 1 3 ⇔ 49 — 8 x 0 = — 3

Очевидно, что действительных корней нет, так как получили отрицательное значение. Делаем вывод, что касательной с углом 150 ° для такой функции не существует.

Вторая функция запишется как

y ‘ = 5 — 49 — 8 x — 4 ‘ = — 1 49 — 8 x ⇒ y ‘ ( x 0 ) = — 1 49 — 8 x 0 = — 1 3 ⇔ 49 — 8 x 0 = — 3 x 0 = 23 4 ⇒ y ( x 0 ) = 5 — 49 — 8 · 23 4 — 4 = — 5 + 3 4

Имеем, что точки касания — 23 4 ; — 5 + 3 4 .

Ответ: уравнение касательной принимает вид

Уравнение касательной

Вы будете перенаправлены на Автор24

Вспомним определение секущей для лучшего понимания что такое касательная.

Секущей называют прямую, пересекающую график кривой в двух точках одновременно.

Касательной прямой к графику кривой называют прямую, проходящую через некую точку кривой и совпадающую с ней в этой точке так, что это прямая лишь касается кривой.

Другое и более ёмкое определение касательной дал Лейбниц.

Лейбниц касательной называл прямую, проведённую через пару точек на рассматриваемой кривой, не совпадающих между собой, но находящихся бесконечно близко друг к другу. Из определения Лейбница видно, что касательная является частным случаем секущей.

Геометрический смысл производной в точке и касательной

Рассмотрим определение касательной подробнее.

Рисунок 1. Касательная и секущая к графику. Автор24 — интернет-биржа студенческих работ

Пусть дана некая кривая $L$, а на ней выбрана произвольная точка $M$. Возьмём ещё одну точку $P$, расположенную также на этой кривой и проведём через точки $M$ и $P$ секущую. Теперь поставим точку $P$ ещё ближе к точке $M$ и проведём новую секущую.

Проделаем так ещё несколько раз, каждый раз получая новую секущую, как бы поворачивающуюся вокруг точки $M$.

В момент, когда очередная точка $P$ находится бесконечно близко к точке $M$, секущая как бы достигает своего предельного положения, в котором по сути она лишь касается графика.

Готовые работы на аналогичную тему

Это положение называется касательной к графику кривой $L$ в точке $M$.

Уравнение касательной через производную

Теперь узнаем, как найти уравнение касательной.

Рассмотрим некую функцию $y(x)$ и выберем на ней точку $M$ с координатами $(a; y(a))$.

Сделаем приращение к аргументу $x$ в этой точке, равное $Δx$ и рассмотрим точку $P$ на графике функции с абсциссой, равной $x=x+Δx$. Значение функции в этой точке будет равно $y(a+ Δx)$. Проведём через точки $M$ и $P$ секущую.

Как мы помним из курса математики, угловой коэффициент равен тангенсу угла прямой с осью абсцисс. Это значит, что угловой коэффициент рассматриваемой нами секущей равен приращению функции $y$ к приращению функции $x$:

Теперь рассмотрим приращение $Δx$ как бесконечно малую величину. В этом случае точка $P$ с координатами $(a; y(a)+ Δy)$ будет приближаться к точке $M$, стремясь к ней. Следовательно, угловой коэффициент нашей секущей, которая в данном случае является касательной, равен пределу:

Воспользуемся формулой $(1)$ для секущей:

Данный предел также носит название производной функции $y=f(x)$ в точке $x$ и обозначается как $y’(x)$.

Геометрический смысл производной состоит в том, что при условии возможности проведения касательной в точке $x$ к графику исследуемой кривой, такой, что эта касательная не параллельна оси $OX$, значение производной является угловым коэффициентом проведённой касательной в этой точке.

Иначе данное утверждение можно записать как

То есть, при составлении уравнения касательной через производную, производная функции является угловым коэффициентом.

Заметим на всякий случай, что сама функция $y=f(x)$ и её производная $y’(x)$ — две разные функции, равные между собой в точке $x$.

Таким образом, в общем виде уравнение касательной будет иметь вид:

где $f(x_0)$ — значение функции в точке $x_0$, а $f’(x_0)$ — её производная.

Уравнение касательной для параболы

Рисунок 2. Уравнение касательной к графику параболической функции. Автор24 — интернет-биржа студенческих работ

Рассмотрим получение уравнения касательной к графику функции на параболе $y=ax^2$ в точке $M$ c координатами $(x; y)$.

Придадим этой точке приращение по оси $OX$, равное $Δx$, приращение по оси $y$ тогда составит $y+Δy=a(x+ Δx)^2$. Точку с координатами $(x+ Δx; y+Δy)$ назовём $P$.

Теперь чтобы определить тангенс угла секущей $MP$с осью абсцисс, рассмотрим прямоугольный треугольник $triangle MNP$. В нём катет $MN$ равен $Δx$, а второй катет $Δy$ — это приращение ординаты, равное $Δy=a(2x cdot Δx + Δx^2)$.

Выразим используя эти данные тангенс угла $φ$.

$mathrm φ=frac =2ax + a cdot Δx$

Теперь для получения углового коэффициента рассмотрим это отношение при бесконечно малой величине $Δx$. Как известно, в этом случае мы имеем дело с пределом:

Благодаря такому соотношению становится легко построить касательную к параболе (рис. 2, б).

Для этого достаточно рассмотреть треугольник $triangle MPT$, так как отрезок $TP$ будет равен:

То есть, для того чтобы получить касательную, необходимо соединить середину отрезка $OP$ с точкой $M$.

Расположение касательной в зависимости от значения её углового коэффициента

Рассмотрим несколько различных случаев значения углового коэффициента для касательной.

Если её угловой коэффициент, то есть, тангенс, равен нулю, то касательная расположена параллельно оси $OX$, а сама прямая принимает вид $y=b$.

Если тангенс положительный, то касательная образует острый угол с осью абсцисс, что значит, что вместе с ростом $x$ растёт и $y$.

В случае если тангенс отрицательный, прямая образует тупой угол с горизонтальной осью, а это значит, что с увеличением значения икса происходит уменьшение значения игрека.

Есть ещё один случай расположения касательной — параллельно оси $OY$, в этом случае её уравнение описывается как $x=c$, где $c$ — некая константа.

Другим числом, определяющим положение касательной, является число $b$, являющееся свободным членом в уравнении прямой $y=kx+b$. Число $b$ характеризует значение функции $y(x)$ в точке её пересечения с осью ординат, иначе говоря, оно есть не что иное, как значение уравнения касательной к графику функции в точке $x=0$.

Составить уравнение касательной в точке $x=3$ для графика функции $y(x)=2x^2+3x-6$.

Сначала найдём значение функции в точке $x=3$:

$y=2 cdot 3^2 +3 cdot 3 – 6 = 21$

Теперь определим значение производной для исследуемой функции:

Теперь получим значение углового коэффициента, для этого подставим $x=3$ в производную:

$y’(x)=4 cdot 3 + 3 = 15$

Подставим это значение в формулу для касательной $(2)$:

$y=15x-24$ — уравнение касательной получено.

Получи деньги за свои студенческие работы

Курсовые, рефераты или другие работы

Автор этой статьи Дата последнего обновления статьи: 14 03 2022

[spoiler title=”источники:”]

http://zaochnik.com/spravochnik/matematika/proizvodnye/kasatelnaja-k-grafiku-funktsii-v-tochke/

http://b4.cooksy.ru/articles/nahozhdenie-kasatelnoy-k-okruzhnosti-cherez-proizvodnuyu

[/spoiler]

Статья дает подробное разъяснение определений, геометрического смысла производной с графическими обозначениями. Будет рассмотрено уравнение касательной прямой с приведением примеров, найдено уравнения касательной к кривым 2 порядка.

Определения и понятия

Определение 1

Угол наклона прямой y=kx+b называется  угол α, который отсчитывается от положительного направления оси ох к прямой y=kx+b в положительном направлении.

Определения и понятия

На рисунке направление ох обозначается при помощи зеленой стрелки и в виде зеленой дуги, а угол наклона при помощи красной дуги. Синяя линия относится к прямой.

Определение 2

Угловой коэффициент прямой y=kx+b называют числовым коэффициентом k.

Угловой коэффициент равняется тангенсу наклона прямой, иначе говоря k=tg α.

  • Угол наклона прямой равняется 0 только при параллельности ох и  угловом коэффициенте, равному нулю, потому как тангенс нуля равен 0. Значит, вид уравнения будет y=b.
  • Если угол наклона прямой y=kx+b острый, тогда выполняются условия 0<α<π2 или 0°<α<90°. Отсюда имеем, что значение углового коэффициента k считается положительным числом, потому как значение тангенс удовлетворяет условию tg α>0, причем имеется возрастание графика.
  • Если α=π2, тогда расположение прямой перпендикулярно ох. Равенство задается при помощи равенства x=c со значением с, являющимся действительным числом.
  • Если угол наклона прямой y=kx+b тупой, то соответствует условиям π2<α<π или 90°<α<180°, значение углового коэффициента k принимает отрицательное значение, а график убывает.
Определение 3

Секущей называют прямую, которая проходит через 2 точки функции f(x). Иначе говоря, секущая – это прямая, которая проводится через любые две точки графика заданной функции.

Определения и понятия

По рисунку видно, что АВ является секущей, а f(x) – черная кривая, α – красная дуга, означающая угол наклона секущей.

Когда угловой коэффициент прямой равняется тангенсу угла наклона, то видно, что тангенс из прямоугольного треугольника АВС можно найти по отношению противолежащего катета к прилежащему.

Определение 4

Получаем формулу для нахождения секущей вида:

k=tg α=BCAC=f(xB)-fxAxB-xA, где абсциссами точек А и В являются значения xA, xB, а f(xA), f(xB) – это значения функции в этих точках.

Очевидно, что угловой коэффициент секущей определен при помощи равенства k=f(xB)-f(xA)xB-xA или k=f(xA)-f(xB)xA-xB, причем уравнение необходимо записать как y=f(xB)-f(xA)xB-xA·x-xA+f(xA) или
y=f(xA)-f(xB)xA-xB·x-xB+f(xB).

Секущая делит график визуально на 3 части: слева от точки А, от А до В, справа от В. На располагаемом ниже рисунке видно, что имеются три секущие, которые считаются совпадающими, то есть задаются при помощи аналогичного уравнения.

Определения и понятия

По определению видно, что прямая и ее секущая в данном случае совпадают.

Секущая может множественно раз пересекать график заданной функции. Если имеется уравнение вида у=0 для секущей, тогда количество точек пересечения с синусоидой бесконечно.

Определение 5

Касательная к графику функции f(x) в точке x0; f(x0) называется прямая, проходящая через заданную точку x0; f(x0),  с наличием отрезка, который имеет множество значений х, близких к x0.

Пример 1

Рассмотрим подробно на ниже приведенном примере. Тогда видно, что прямая, заданная функцией y=x+1, считается касательной к y=2x в точке  с координатами (1; 2). Для наглядности, необходимо рассмотреть графики с приближенными к (1; 2) значениями. Функция y=2x обозначена черным цветом, синяя линия – касательная, красная точка – точка пересечения.

Определения и понятия

Очевидно, что y=2x сливается с прямой у=х+1.

Для определения касательной следует рассмотреть поведение касательной АВ при бесконечном приближении точки В к точке А. Для наглядности приведем рисунок.

Определения и понятия

Секущая АВ, обозначенная при помощи синей линии, стремится к положению самой касательной, а угол наклона секущей α начнет стремиться к углу наклона самой касательной αx.

Определение 6

Касательной к графику функции y=f(x) в точке А считается предельное положение секущей АВ при В стремящейся к А, то есть B→A.

Теперь перейдем к рассмотрению геометрического смысла производной функции в точке.

Геометрический смысл производной функции в точке

Перейдем к рассмотрению секущей АВ для функции f(x), где А и В с координатами x0, f(x0) и x0+∆x, f(x0+∆x), а ∆x обозначаем как приращение аргумента. Теперь функция примет вид ∆y=∆f(x)=f(x0+∆x)-f(∆x). Для наглядности приведем в пример рисунок.

Геометрический смысл производной функции в точке

Рассмотрим полученный прямоугольный треугольник АВС. Используем определение тангенса для решения, то есть получим отношение ∆y∆x=tg α. Из определения касательной следует, что lim∆x→0∆y∆x=tg αx. По правилу производной в точке имеем, что производную f(x) в точке x0 называют пределом отношений приращения функции к приращению аргумента, где ∆x→0, тогда обозначим как f(x0)=lim∆x→0∆y∆x.

Отсюда следует, что f'(x0)=lim∆x→0∆y∆x=tg αx=kx, где kx обозначают в качестве углового коэффициента касательной.

То есть получаем, что f’(x) может существовать  в точке x0 причем как и касательная к заданному графику функции в точке касания равной x0, f0(x0), где значение углового коэффициента касательной  в точке равняется производной  в точке x0. Тогда получаем, что kx=f'(x0).

Геометрический смысл производной функции в точке в том, что дается понятие существования касательной к графику в этой же точке.

Уравнение касательной прямой

Чтобы записать уравнение любой прямой на плоскости, необходимо иметь угловой коэффициент с точкой, через которую она проходит. Его обозначение принимается как x0 при пересечении.

Уравнение касательной к графику функции y=f(x) в точке x0, f0(x0) принимает вид y=f'(x0)·x-x0+f(x0).

Имеется в виду, что конечным значением производной f'(x0) можно определить положение касательной, то есть вертикально при условии limx→x0+0f'(x)=∞ и limx→x0-0f'(x)=∞ или отсутствие вовсе при условии limx→x0+0f'(x)≠limx→x0-0f'(x).

Расположение касательной зависит от значения ее углового коэффициента kx=f'(x0). При параллельности к оси ох получаем, что kk=0, при параллельности к оу – kx=∞, причем вид уравнения касательной x=x0 возрастает при kx>0, убывает при kx<0.

Пример 2

Произвести составление уравнения касательной к графику функции y=ex+1+x33-6-33x-17-33 в точке  с координатами (1; 3) с определением угла наклона.

Решение

По условию имеем, что функция определяется для всех действительных чисел. Получаем, что точка с координатами, заданными по условию, (1; 3) является точкой касания, тогда x0=-1, f(x0)=-3.

Необходимо найти производную в точке со значением -1. Получаем, что

y’=ex+1+x33-6-33x-17-33’==ex+1’+x33′-6-33x’-17-33’=ex+1+x2-6-33y'(x0)=y'(-1)=e-1+1+-12-6-33=33

Значение f’(x) в точке касания является  угловым коэффициентом касательной, который равняется тангенсу наклона.

Тогда kx=tg αx=y'(x0)=33

Отсюда следует, что αx=arctg33=π6

Ответ: уравнение касательной приобретает вид

y=f'(x0)·x-x0+f(x0)y=33(x+1)-3y=33x-9-33

Для наглядности приведем пример в графической иллюстрации.

Черный цвет используется для графика исходной функции, синий цвет – изображение касательной, красная точка – точка касания. Рисунок, располагаемый справа, показывает  в увеличенном виде.

Уравнение касательной прямой

Пример 3

Выяснить наличие существования касательной к графику заданной функции
y=3·x-15+1 в точке с координатами (1;1). Составить уравнение и определить угол наклона.

Решение

По условию имеем, что областью определения заданной функции считается множество всех действительных чисел.

Перейдем к нахождению производной

y’=3·x-15+1’=3·15·(x-1)15-1=35·1(x-1)45

Если x0=1, тогда f’(x) не определена, но пределы записываются как  limx→1+035·1(x-1)45=35·1(+0)45=35·1+0=+∞ и limx→1-035·1(x-1)45=35·1(-0)45=35·1+0=+∞, что означает существование вертикальной касательной в точке (1;1).

Ответ: уравнение примет вид х=1, где угол наклона будет равен π2.

Для наглядности изобразим графически.

Уравнение касательной прямой

Пример 4

Найти точки графика функции y=115x+23-45×2-165x-265+3x+2, где

  1. Касательная не существует;
  2. Касательная располагается параллельно ох;
  3. Касательная параллельна прямой y=85x+4.

Решение

Необходимо обратить внимание на область определения. По условию имеем, что функция определена на множестве всех действительных чисел. Раскрываем модуль и решаем систему с промежутками x∈-∞; 2 и [-2; +∞). Получаем, что

y=-115×3+18×2+105x+176, x∈-∞; -2115×3-6×2+9x+12, x∈[-2; +∞)

Необходимо продифференцировать функцию. Имеем, что

y’=-115×3+18×2+105x+176′, x∈-∞; -2115×3-6×2+9x+12′, x∈[-2; +∞)⇔y’=-15(x2+12x+35), x∈-∞; -215×2-4x+3, x∈[-2; +∞)

Когда х=-2, тогда производная не существует, потому что односторонние пределы не равны в этой точке:

limx→-2-0y'(x)=limx→-2-0-15(x2+12x+35=-15(-2)2+12(-2)+35=-3limx→-2+0y'(x)=limx→-2+015(x2-4x+3)=15-22-4-2+3=3

Вычисляем значение функции в точке х=-2, где получаем, что

  1. y(-2)=115-2+23-45(-2)2-165(-2)-265+3-2+2=-2, то есть касательная в точке (-2;-2) не будет существовать.
  2. Касательная параллельна ох, когда угловой коэффициент равняется нулю. Тогда kx=tg αx=f'(x0). То есть необходимо найти значения таких х, когда производная функции  обращает ее в ноль. То есть значения f’(x) и будут являться точками касания, где касательная является параллельной ох.

Когда x∈-∞; -2, тогда -15(x2+12x+35)=0, а при x∈(-2; +∞) получаем 15(x2-4x+3)=0.

Решим:

-15(x2+12x+35)=0D=122-4·35=144-140=4×1=-12+42=-5∈-∞; -2×2=-12-42=-7∈-∞; -2   15(x2-4x+3)=0D=42-4·3=4×3=4-42=1∈-2; +∞x4=4+42=3∈-2; +∞

Вычисляем соответствующие значения функции

y1=y-5=115-5+23-45-52-165-5-265+3-5+2=85y2=y(-7)=115-7+23-45(-7)2-165-7-265+3-7+2=43y3=y(1)=1151+23-45·12-165·1-265+31+2=85y4=y(3)=1153+23-45·32-165·3-265+33+2=43

Отсюда -5; 85, -4; 43, 1; 85, 3; 43 считаются искомыми точками графика функции.

Рассмотрим графическое изображение решения.

Уравнение касательной прямой

Черная линия – график функции, красные точки – точки касания.

  1. Когда прямые располагаются параллельно, то угловые коэффициенты равны. Тогда необходимо заняться поиском точек графика функции, где угловой коэффициент будет равняться значению 85 . Для этого нужно решить уравнение вида y'(x)=85. Тогда, если x∈-∞; -2, получаем, что -15(x2+12x+35)=85, а если x∈(-2; +∞), тогда 15(x2-4x+3)=85.

Первое уравнение не имеет корней, так как дискриминант меньше нуля. Запишем, что

-15×2+12x+35=85×2+12x+43=0D=122-4·43=-28<0

Другое уравнение имеет два действительных корня, тогда

15(x2-4x+3)=85×2-4x-5=0D=42-4·(-5)=36×1=4-362=-1∈-2; +∞x2=4+362=5∈-2; +∞

Перейдем к нахождению значений функции. Получаем, что

y1=y(-1)=115-1+23-45(-1)2-165(-1)-265+3-1+2=415y2=y(5)=1155+23-45·52-165·5-265+35+2=83

Точки со значениями -1; 415, 5; 83 являются точками, в которых касательные параллельны прямой y=85x+4.

Ответ: черная линия – график функции, красная линия – график y=85x+4, синяя линия – касательные  в точках -1; 415, 5; 83.

Уравнение касательной прямой

Возможно существование бесконечного количества касательных для заданных функций.

Пример 5

Написать уравнения всех имеющихся касательных функции y=3cos32x-π4-13, которые располагаются перпендикулярно прямой y=-2x+12.

Решение

Для составления уравнения касательной необходимо найти коэффициент и координаты точки касания, исходя из условия перпендикулярности прямых. Определение звучит так: произведение угловых коэффициентов, которые перпендикулярны прямым, равняется -1, то есть записывается как kx·k⊥=-1. Из условия имеем, что угловой коэффициент располагается перпендикулярно прямой  и равняется k⊥=-2, тогда kx=-1k⊥=-1-2=12.

Теперь необходимо найти координаты точек касания. Нужно найти х, после чего его значение для заданной функции. Отметим, что из геометрического смысла производной  в точке
x0 получаем, что kx=y'(x0).  Из данного равенства найдем значения х для точек касания.

Получаем, что

y'(x0)=3cos32x0-π4-13’=3·-sin32x0-π4·32×0-π4’==-3·sin32x0-π4·32=-92·sin32x0-π4⇒kx=y'(x0)⇔-92·sin32x0-π4=12⇒sin32x0-π4=-19

Это тригонометрическое уравнение будет использовано для вычисления ординат точек касания.

32×0-π4=arcsin-19+2πk или 32×0-π4=π-arcsin-19+2πk

32×0-π4=-arcsin19+2πk или 32×0-π4=π+arcsin19+2πk

x0=23π4-arcsin19+2πk или x0=235π4+arcsin19+2πk, k∈Z

Z- множество целых чисел.

Найдены х точек касания. Теперь необходимо перейти к поиску значений у:

y0=3cos32x0-π4-13

y0=3·1-sin232x0-π4-13 или y0=3·-1-sin232x0-π4-13

y0=3·1–192-13 или y0=3·-1–192-13

y0=45-13 или y0=-45+13

Отсюда получаем, что 23π4-arcsin19+2πk; 45-13, 235π4+arcsin19+2πk; -45+13 являются точками касания.

Ответ: необходимы уравнения запишутся как

y=12x-23π4-arcsin19+2πk+45-13,y=12x-235π4+arcsin19+2πk-45+13, k∈Z

Для наглядного изображения рассмотрим функцию и касательную на координатной прямой.

Рисунок показывает, что расположение функции идет на промежутке [-10;10], где черная прямя – график функции, синие линии – касательные, которые располагаются перпендикулярно заданной прямой вида y=-2x+12. Красные точки – это точки касания.

Уравнение касательной прямой

Касательная к окружности, эллипсу, гиперболе, параболе

Канонические уравнения кривых 2 порядка не являются однозначными функциями. Уравнения касательных для них составляются по известным схемам.

Касательная к окружности

Для задания окружности  с центром  в точке xcenter; ycenter и радиусом R применяется формула x-xcenter2+y-ycenter2=R2.

Данное равенство может быть записано как объединение двух функций:

y=R2-x-xcenter2+ycentery=-R2-x-xcenter2+ycenter

Первая функция располагается вверху, а вторая внизу, как показано на рисунке.

Касательная к окружности, эллипсу, гиперболе, параболе

Для составления уравнения окружности  в точке x0; y0, которая располагается  в верхней или нижней полуокружности, следует найти уравнение графика функции вида y=R2-x-xcenter2+ycenter или y=-R2-x-xcenter2+ycenter в указанной точке.

Когда в точках xcenter; ycenter+R и xcenter; ycenter-R касательные могут быть заданы уравнениями y=ycenter+R и y=ycenter-R, а  в точках xcenter+R; ycenter и
xcenter-R; ycenter будут являться параллельными оу, тогда получим уравнения вида x=xcenter+R и x=xcenter-R.

Касательная к окружности, эллипсу, гиперболе, параболе

Касательная к эллипсу

Когда эллипс имеет центр  в точке xcenter; ycenter с полуосями a и b, тогда он может быть задан при помощи уравнения x-xcenter2a2+y-ycenter2b2=1.

Эллипс и окружность могут быть обозначаться при помощи объединения двух функций, а именно: верхнего и нижнего полуэллипса. Тогда получаем, что

y=ba·a2-(x-xcenter)2+ycentery=-ba·a2-(x-xcenter)2+ycenter

Касательная к окружности, эллипсу, гиперболе, параболе

Если  касательные располагаются на вершинах эллипса, тогда они параллельны ох или оу. Ниже для наглядности рассмотрим рисунок.

Касательная к окружности, эллипсу, гиперболе, параболе

Пример 6

Написать уравнение касательной к эллипсу x-324+y-5225=1 в точках со значениями x равного х=2.

Решение

Необходимо найти точки касания, которые соответствуют значению х=2. Производим подстановку в имеющееся уравнение эллипса и получаем, что

x-324x=2+y-5225=114+y-5225=1⇒y-52=34·25⇒y=±532+5

Тогда 2; 532+5 и 2; -532+5 являются точками касания, которые принадлежат верхнему и нижнему полуэллипсу.

Перейдем к нахождению и разрешению уравнения эллипса относительно y. Получим, что

x-324+y-5225=1y-5225=1-x-324(y-5)2=25·1-x-324y-5=±5·1-x-324y=5±524-x-32

Очевидно, что верхний полуэллипс задается с помощью функции вида y=5+524-x-32, а нижний y=5-524-x-32.

Применим стандартный алгоритм для того, чтобы составить уравнение касательной к графику функции в точке. Запишем, что уравнение для первой касательной в точке 2; 532+5 будет иметь вид

y’=5+524-x-32’=52·124-(x-3)2·4-(x-3)2’==-52·x-34-(x-3)2⇒y'(x0)=y'(2)=-52·2-34-(2-3)2=523⇒y=y'(x0)·x-x0+y0⇔y=523(x-2)+532+5

Получаем, что уравнение второй касательной со значением в точке
2; -532+5 принимает вид

y’=5-524-(x-3)2’=-52·124-(x-3)2·4-(x-3)2’==52·x-34-(x-3)2⇒y'(x0)=y'(2)=52·2-34-(2-3)2=-523⇒y=y'(x0)·x-x0+y0⇔y=-523(x-2)-532+5

Графически касательные обозначаются  так:

Касательная к окружности, эллипсу, гиперболе, параболе

Касательная к гиперболе

Когда гипербола имеет центр в точке xcenter; ycenter и вершины xcenter+α; ycenter и xcenter-α; ycenter, имеет место задание неравенства x-xcenter2α2-y-ycenter2b2=1, если с вершинами xcenter; ycenter+b и xcenter; ycenter-b, тогда задается при помощи неравенства x-xcenter2α2-y-ycenter2b2=-1.

Касательная к окружности, эллипсу, гиперболе, параболе

Гипербола может быть представлена в виде двух объединенных функций вида

y=ba·(x-xcenter)2-a2+ycentery=-ba·(x-xcenter)2-a2+ycenter или y=ba·(x-xcenter)2+a2+ycentery=-ba·(x-xcenter)2+a2+ycenter

Касательная к окружности, эллипсу, гиперболе, параболе

В первом случае имеем, что касательные параллельны оу, а во втором параллельны ох.

Отсюда следует, что для того, чтобы найти уравнение касательной к гиперболе, необходимо выяснить, какой функции принадлежит точка касания. Чтобы определить это, необходимо произвести подстановку в уравнения и проверить их на тождественность.

Пример 7

Составить уравнение касательной к гиперболе x-324-y+329=1 в точке 7; -33-3.

Решение

Необходимо преобразовать запись решения нахождения гиперболы при помощи 2 функций. Получим, что

x-324-y+329=1⇒y+329=x-324-1⇒y+32=9·x-324-1⇒y+3=32·x-32-4 или y+3=-32·x-32-4⇒y=32·x-32-4-3y=-32·x-32-4-3

Необходимо выявить, к какой функции принадлежит заданная точка с координатами 7; -33-3.

Очевидно, что для проверки первой функции необходимо y(7)=32·(7-3)2-4-3=33-3≠-33-3, тогда точка графику не принадлежит, так как равенство не выполняется.

Для второй функции имеем, что y(7)=-32·(7-3)2-4-3=-33-3≠-33-3, значит, точка принадлежит заданному графику. Отсюда следует найти угловой коэффициент.

Получаем, что

y’=-32·(x-3)2-4-3’=-32·x-3(x-3)2-4⇒kx=y'(x0)=-32·x0-3×0-32-4×0=7=-32·7-37-32-4=-3

Ответ: уравнение касательной можно представить как

y=-3·x-7-33-3=-3·x+43-3

Наглядно изображается так:

Касательная к окружности, эллипсу, гиперболе, параболе

Касательная к параболе

Чтобы составить уравнение касательной к параболе y=ax2+bx+c в точке x0, y(x0), необходимо использовать стандартный алгоритм, тогда уравнение примет вид y=y'(x0)·x-x0+y(x0). Такая касательная в вершине параллельна ох.

Следует задать параболу x=ay2+by+c как объединение двух функций. Поэтому нужно разрешить уравнение относительно у. Получаем, что

x=ay2+by+c⇔ay2+by+c-x=0D=b2-4a(c-x)y=-b+b2-4a(c-x)2ay=-b-b2-4a(c-x)2a

Графически изобразим как:

Касательная к окружности, эллипсу, гиперболе, параболе

Для выяснения принадлежности точки x0, y(x0) функции, нежно действовать по стандартному алгоритму. Такая касательная будет параллельна оу относительно параболы.

Пример 8

Написать уравнение касательной к графику x-2y2-5y+3, когда имеем угол наклона касательной 150°.

Решение

Начинаем решение с представления параболы в качестве двух функций. Получим, что

-2y2-5y+3-x=0D=(-5)2-4·(-2)·(3-x)=49-8xy=5+49-8x-4y=5-49-8x-4

Значение углового коэффициента равняется значению производной в точке x0 этой функции и равняется тангенсу угла наклона.

Получаем:

kx=y'(x0)=tg αx=tg 150°=-13

Отсюда определим значение х для точек касания.

Первая функция запишется как

y’=5+49-8x-4’=149-8x⇒y'(x0)=149-8×0=-13⇔49-8×0=-3

Очевидно, что действительных корней нет, так как получили отрицательное значение. Делаем вывод, что касательной с углом 150° для такой функции не существует.

Вторая функция запишется как

y’=5-49-8x-4’=-149-8x⇒y'(x0)=-149-8×0=-13⇔49-8×0=-3×0=234⇒y(x0)=5-49-8·234-4=-5+34

Имеем, что точки касания – 234; -5+34.

Ответ: уравнение касательной принимает вид

y=-13·x-234+-5+34

Графически изобразим это таким образом:

Касательная к окружности, эллипсу, гиперболе, параболе

Параметрическая касательная к окружности (задача за 9 класс)

Приветствую вас, друзья! В этой заметке подробно разберем задачу из ОГЭ по математике. Получается, что уровень сложности задачи: ~ 9 класс.

Задание

Прямая y  =  2•x + b касается окружности x²  +  y² =  5 в точке с положительной абсциссой. Определите координаты точки касания.

Решение:

В самом начале я привел рисунок, который описывает как может располагаться прямая и окружности (рисунок не подходит к нашей задаче, это сделано специально для того, чтобы бы вы подумали на начальном этапе).

Итак, прямая может:
1. Не иметь с окружностью общих точек (не пересекаться)
2. Иметь с окружностью одну общую точку (касаться окружности)
3. Иметь с окружностью две общих точкий (пересекать окружность, образуя хорду).

В нашей задаче нужно рассмотреть именно 2-й случай (обратите внимание на условие задачи).

Способ 1

Допустим, нам необходимо решить задачу со знаниями 9 класса. Найдем точку пересечения функций. Для этого решим совместную систему:

Параметрическая касательная к окружности (задача за 9 класс)

Решение последнего уравнения определяет количество точек пересечения прямой с окружностью и координаты этих точек. Так как в задании сказано, что прямая является касательной, то квадратное уравнение должно давать одно решение, зависящее от параметра. То есть в нашем уравнении должен быть нулевой дискриминант. Учтем это:

Параметрическая касательная к окружности (задача за 9 класс)

Если мы представим графическое решение, то поймем, что касание может происходит сверху и снизу. Нам же нужно выбрать ту точку, у которой будет положительное значение абсциссы.

Ответ: x₀ = 2 при значении параметра b = – 5

Способ 2

Если мы уже знакомы с производными, то можно написать уравнение для касательной к окружности. Анализ функций дает нам подсказку, что касание должно происходить в области, где x > 0 и y < 0, это значит, что функцию можем выразить явно. Это может понадобиться для дальнейшего нахождения производной этой функции. Производная в точке касания будет определять коэффициент наклона касательной.

Параметрическая касательная к окружности (задача за 9 класс)

Данное уравнение совпадает с уравнением касательной в условии задачи. Поэтому, приравняв соответствующие коэффициенты, мы сможем найти как абсциссу касания, так и значения параметра:

Параметрическая касательная к окружности (задача за 9 класс)

Выбираем положительное значение, получаем тот же ответ:

Ответ: x₀ = 2 при значении параметра b = – 5

Общий ответ: точка касания M(2; -1) при значение параметра b = -5

А теперь правильный график, иллюстрирующий касание параметрической прямой к окружности
А теперь правильный график, иллюстрирующий касание параметрической прямой к окружности

Понравилась заметка ? Поставьте лайк, подпишитесь на канал! Вам не сложно, а мне очень приятно 🙂

Если Вам нужен репетитор по физике, математике или информатике/программированию, Вы можете написать мне или в мою группу Репетитор IT mentor в VK
Библиотека с книгами для физиков, математиков и программистов
Репетитор IT mentor в telegram

Вы уже знаете, какую прямую называют касательной к окружности. А что понимают, например, под касательной к синусоиде? Прямая Касательная к графику функции и производная с примерами решения

Пусть даны график функции Касательная к графику функции и производная с примерами решения и на ней точка Касательная к графику функции и производная с примерами решения которая не является концом графика (рис. 60). Обозначим на данном графике по разные стороны от Касательная к графику функции и производная с примерами решения произвольные точки Касательная к графику функции и производная с примерами решения Прямые Касательная к графику функции и производная с примерами решения — секущие. Если же точки Касательная к графику функции и производная с примерами решения двигаясь по графику, приближать достаточно близко к Касательная к графику функции и производная с примерами решения как угодно близко будут приближаться к некоторой прямой Касательная к графику функции и производная с примерами решения Такую прямую Касательная к графику функции и производная с примерами решения (если она существует) называют касательной к графику функции Касательная к графику функции и производная с примерами решения в точке Касательная к графику функции и производная с примерами решения

Если график функции такой, как показано на рисунке 61, то при неограниченном приближении точек Касательная к графику функции и производная с примерами решения к точке Касательная к графику функции и производная с примерами решения предельные положения секущих — прямые Касательная к графику функции и производная с примерами решения — не совпадут. Говорят, что в точке Касательная к графику функции и производная с примерами решения касательной к графику функции  не существует.

Касательная к графику функции и производная с примерами решения

Касательная к графику функции и производная с примерами решения

И если Касательная к графику функции и производная с примерами решения — крайняя точка графика, то касательной к нему в точке Касательная к графику функции и производная с примерами решения не существует.

Понятие касательной к графику часто используют для исследования функций. Рассмотрим этот вопрос сначала в общем виде.

Касательная — это прямая. Её уравнение имеет вид Касательная к графику функции и производная с примерами решения где Касательная к графику функции и производная с примерами решения — угловой коэффициент — тангенс угла между лучом касательной, расположенным выше оси Касательная к графику функции и производная с примерами решения и положительным направлением этой оси. Обратите внимание на угловой коэффициент Касательная к графику функции и производная с примерами решения касательной, проведённой к графику какой-либо функции в его точке с абсциссой Касательная к графику функции и производная с примерами решения Если число Касательная к графику функции и производная с примерами решения принадлежит промежутку возрастания функции, то соответствующее значение Касательная к графику функции и производная с примерами решения положительное (рис. 62). Если Касательная к графику функции и производная с примерами решения принадлежит промежутку убывания функции, то Касательная к графику функции и производная с примерами решения — отрицательное (рис. 63). И наоборот: если каждому значению Касательная к графику функции и производная с примерами решения из некоторого промежутка Касательная к графику функции и производная с примерами решения соответствует положительное значение Касательная к графику функции и производная с примерами решения то на Касательная к графику функции и производная с примерами решения данная функция возрастает; если каждому значению Касательная к графику функции и производная с примерами решения из некоторого промежутка Касательная к графику функции и производная с примерами решения соответствует отрицательное значение Касательная к графику функции и производная с примерами решения то на  функция убывает. Заслуживают внимания и те точки графика функции, в которых касательная не существует, и в которых она параллельна оси Касательная к графику функции и производная с примерами решения

Касательная к графику функции и производная с примерами решения

Итак, зная угловые коэффициенты касательных к графику функции в тех или иных точках, можно сделать вывод, возрастает данная функция в этих точках, или убывает.

Поскольку для исследования функций важно уметь определять угловой коэффициент касательной к её графику, то рассмотрим подробнее связь этого коэффициента с исследуемой функцией.

Пусть даны график функции Касательная к графику функции и производная с примерами решения и на ней точку Касательная к графику функции и производная с примерами решения в которой существует касательная к графику (рис. 64). Если абсцисса точки Касательная к графику функции и производная с примерами решения равна Касательная к графику функции и производная с примерами решения то её ордината — Касательная к графику функции и производная с примерами решения Дадим значению аргумента Касательная к графику функции и производная с примерами решения приращение Касательная к графику функции и производная с примерами решения Тогда значению аргумента Касательная к графику функции и производная с примерами решения на графике функции соответствует точка Касательная к графику функции и производная с примерами решения с абсциссой Касательная к графику функции и производная с примерами решения и ординатой Касательная к графику функции и производная с примерами решения

Касательная к графику функции и производная с примерами решения

Через точки Касательная к графику функции и производная с примерами решения проведём прямые Касательная к графику функции и производная с примерами решения параллельные осям абсцисс и ординат. Они пересекутся в некоторой точке Касательная к графику функции и производная с примерами решения Тогда Касательная к графику функции и производная с примерами решения — приращение аргумента, а Касательная к графику функции и производная с примерами решения — приращение функции на Касательная к графику функции и производная с примерами решения

Угловой коэффициент секущей Касательная к графику функции и производная с примерами решения равен тангенсу угла Касательная к графику функции и производная с примерами решения т. е. отношению Касательная к графику функции и производная с примерами решения

Касательная к графику функции и производная с примерами решения

Если Касательная к графику функции и производная с примерами решения то секущая Касательная к графику функции и производная с примерами решения поворачиваясь вокруг точки Касательная к графику функции и производная с примерами решения приближается к касательной, проведённой в точке Касательная к графику функции и производная с примерами решения к графику данной функции. Итак, если Касательная к графику функции и производная с примерами решения — угловой коэффициент этой касательной и Касательная к графику функции и производная с примерами решения то

Касательная к графику функции и производная с примерами решения

Так определяется угловой коэффициент касательной к графику функции Касательная к графику функции и производная с примерами решения в некоторой точке Касательная к графику функции и производная с примерами решения если касательная в ней не параллельна оси Касательная к графику функции и производная с примерами решения Если касательная к графику функции в некоторой точке параллельна оси Касательная к графику функции и производная с примерами решения то угловой коэффициент этой касательной равен нулю.

К вычислению значения выражения Касательная к графику функции и производная с примерами решения  или Касательная к графику функции и производная с примерами решения приводит решение многих задач по механике, электричеству, биологии, экономике, статистике и т. д. Именно поэтому это выражение получило специальное название — производная.

Производной функции Касательная к графику функции и производная с примерами решения в точке Касательная к графику функции и производная с примерами решения называют предел отношения приращения функции в точке Касательная к графику функции и производная с примерами решения к приращению аргумента, если приращение аргумента стремится к нулю, а предел существует.

Производную функции Касательная к графику функции и производная с примерами решения в точке Касательная к графику функции и производная с примерами решения обозначают Касательная к графику функции и производная с примерами решения Её определение записывают также в виде равенства:

Касательная к графику функции и производная с примерами решения

Пример:

Найдите производную функции Касательная к графику функции и производная с примерами решения в точке Касательная к графику функции и производная с примерами решения

Решение:

Дадим аргументу Касательная к графику функции и производная с примерами решения приращение Касательная к графику функции и производная с примерами решения Соответствующее приращение функции Касательная к графику функции и производная с примерами решения

Тогда Касательная к графику функции и производная с примерами решения Если Касательная к графику функции и производная с примерами решения

Следовательно, Касательная к графику функции и производная с примерами решения

Ответ. Касательная к графику функции и производная с примерами решения

Так решают задачу, пользуясь определением производной функции в точке.

До сих пор речь шла о производной функции в точке. А можно рассматривать производную функции и как функцию. Пусть, например, дана функция Касательная к графику функции и производная с примерами решенияНайдём её производную в произвольной точке Касательная к графику функции и производная с примерами решения Для этого дадим значению Касательная к графику функции и производная с примерами решенияприращение Касательная к графику функции и производная с примерами решения Соответствующее ему приращение функции

Касательная к графику функции и производная с примерами решения

Поэтому Касательная к графику функции и производная с примерами решения Если Касательная к графику функции и производная с примерами решения

Имеем Касательная к графику функции и производная с примерами решения

Следовательно, производная функции Касательная к графику функции и производная с примерами решения в каждой её точке Касательная к графику функции и производная с примерами решения равна Касательная к графику функции и производная с примерами решения Пишут: Касательная к графику функции и производная с примерами решения или, если Касательная к графику функции и производная с примерами решения

Обратите внимание! Производная функции в точке — это число. Когда же говорят о производной, не указывая «в точке», подразумевают производную как функцию: производной функции Касательная к графику функции и производная с примерами решения есть функция Касательная к графику функции и производная с примерами решения производной функции Касательная к графику функции и производная с примерами решения есть функция Касательная к графику функции и производная с примерами решения и т. д.

Зная это, производную функции в точке можно вычислять проще, чем по определению производной функции в точке. Пример 2. Дана функция Касательная к графику функции и производная с примерами решенияНайдите Касательная к графику функции и производная с примерами решения Решение. Производной функции Касательная к графику функции и производная с примерами решения является функция Касательная к графику функции и производная с примерами решения Поэтому Касательная к графику функции и производная с примерами решенияКасательная к графику функции и производная с примерами решения

 Нахождение производной называется дифференцированием.  Функция, которая имеет производную в точке Касательная к графику функции и производная с примерами решения называется дифференцируемой в точке Касательная к графику функции и производная с примерами решения Функция, дифференцируемая в каждой точке некоторого промежутка, называется дифференцируемой на этом промежутке.

Докажем, например, что линейная функция Касательная к графику функции и производная с примерами решения дифференцируема в каждой точке Касательная к графику функции и производная с примерами решения Действительно, приращению Касательная к графику функции и производная с примерами решения её аргумента Касательная к графику функции и производная с примерами решения соответствует приращение функции Касательная к графику функции и производная с примерами решения Поэтому Касательная к графику функции и производная с примерами решения и если Касательная к графику функции и производная с примерами решения А это и значит, что в каждой точке Касательная к графику функции и производная с примерами решения функция Касательная к графику функции и производная с примерами решения имеет производную Касательная к графику функции и производная с примерами решения

 Пишут Касательная к графику функции и производная с примерами решения

 В частности: Касательная к графику функции и производная с примерами решения

 Производная постоянной равна нулю.

Из курса планиметрии известно, что уравнение прямой, проходящей через заданную точку Касательная к графику функции и производная с примерами решения имеет вид Касательная к графику функции и производная с примерами решения где Касательная к графику функции и производная с примерами решения — угловой коэффициент прямой.

Поскольку для касательной к графику функции Касательная к графику функции и производная с примерами решения угловой коэффициент равен значению производной в точке касания Касательная к графику функции и производная с примерами решения то можем записать общий вид уравнения касательной, проведённой к графику функции Касательная к графику функции и производная с примерами решения в точке касания Касательная к графику функции и производная с примерами решения

Касательная к графику функции и производная с примерами решения

До сих пор речь шла о касательных к криволинейным графикам. Но графиком функции может быть и прямая или часть прямой. Поэтому для обобщения договариваются касательной к прямой в любой её точке считать эту самую прямую. Касательной к отрезку или лучу в любой его внутренней точке считают прямую, которой принадлежит этот отрезок или луч.

Выше было установлено, что производная линейной функции равна коэффициенту при переменной, т.е Касательная к графику функции и производная с примерами решения

Полученный результат имеет очевидный геометрический смысл: касательная к прямой — графику функции Касательная к графику функции и производная с примерами решения — есть эта самая прямая, её угловой коэффициент равен Касательная к графику функции и производная с примерами решения

  • Заказать решение задач по высшей математике

Пример:

Найдите угол, который образуете положительным направлением оси Касательная к графику функции и производная с примерами решениякасательная к графику функции Касательная к графику функции и производная с примерами решения в точке Касательная к графику функции и производная с примерами решения

Решение:

Определим сначала угловой коэффициент этой касательной по формуле Касательная к графику функции и производная с примерами решения — приращения функции и приращения аргумента соответственно.

Найдем приращение функции Касательная к графику функции и производная с примерами решения в точке Касательная к графику функции и производная с примерами решения

Касательная к графику функции и производная с примерами решения

Найдём угловой коэффициент касательной:

Касательная к графику функции и производная с примерами решения

Поскольку Касательная к графику функции и производная с примерами решения

Известно также, что Касательная к графику функции и производная с примерами решения поэтому Касательная к графику функции и производная с примерами решения отсюда Касательная к графику функции и производная с примерами решения

Пример:

Докажите, что для функции Касательная к графику функции и производная с примерами решения производной есть функция Касательная к графику функции и производная с примерами решения

Решение:

 Касательная к графику функции и производная с примерами решенияКасательная к графику функции и производная с примерами решения Если Касательная к графику функции и производная с примерами решения А это и означает, что производной функции Касательная к графику функции и производная с примерами решения является функция Касательная к графику функции и производная с примерами решения

Пример:

Напишите уравнение касательной к графику функции Касательная к графику функции и производная с примерами решения в его точке с абсциссой Касательная к графику функции и производная с примерами решения

Решение:

Способ 1. Уравнение касательной имеет вид Касательная к графику функции и производная с примерами решения Угловой коэффициент Касательная к графику функции и производная с примерами решения равен значению производной функции Касательная к графику функции и производная с примерами решения в точке Касательная к графику функции и производная с примерами решения Значит, уравнение касательной Касательная к графику функции и производная с примерами решенияКоординаты точки касания Касательная к графику функции и производная с примерами решения Точка с такими координатами принадлежит касательной, поэтому Касательная к графику функции и производная с примерами решения отсюда Касательная к графику функции и производная с примерами решенияСледовательно, уравнение касательной имеет вид: Касательная к графику функции и производная с примерами решения

Способ 2. Запишем общий вид уравнения касательной:

Касательная к графику функции и производная с примерами решения

Найдём Касательная к графику функции и производная с примерами решения

Касательная к графику функции и производная с примерами решения

Подставим найденные значения в уравнение касательной:

Касательная к графику функции и производная с примерами решения

  • Предел и непрерывность функции
  • Свойства функций, непрерывных в точке и на промежутке
  • Предел функции на бесконечности
  • Применение производной к исследованию функции
  • Иррациональные неравенства
  • Производная в математике
  • Как найти производную функции
  • Асимптоты графика функции
Автор статьи

Наталья Игоревна Восковская

Эксперт по предмету «Математика»

Задать вопрос автору статьи

Вспомним определение секущей для лучшего понимания что такое касательная.

Определение 1

Секущей называют прямую, пересекающую график кривой в двух точках одновременно.

Касательной прямой к графику кривой называют прямую, проходящую через некую точку кривой и совпадающую с ней в этой точке так, что это прямая лишь касается кривой.

Другое и более ёмкое определение касательной дал Лейбниц.

Определение 2

Лейбниц касательной называл прямую, проведённую через пару точек на рассматриваемой кривой, не совпадающих между собой, но находящихся бесконечно близко друг к другу. Из определения Лейбница видно, что касательная является частным случаем секущей.

Логотип baranka

Сдай на права пока
учишься в ВУЗе

Вся теория в удобном приложении. Выбери инструктора и начни заниматься!

Получить скидку 3 000 ₽

Геометрический смысл производной в точке и касательной

Рассмотрим определение касательной подробнее.

Касательная и секущая к графику. Автор24 — интернет-биржа студенческих работ

Рисунок 1. Касательная и секущая к графику. Автор24 — интернет-биржа студенческих работ

Пусть дана некая кривая $L$, а на ней выбрана произвольная точка $M$. Возьмём ещё одну точку $P$, расположенную также на этой кривой и проведём через точки $M$ и $P$ секущую. Теперь поставим точку $P$ ещё ближе к точке $M$ и проведём новую секущую.

Проделаем так ещё несколько раз, каждый раз получая новую секущую, как бы поворачивающуюся вокруг точки $M$.

В момент, когда очередная точка $P$ находится бесконечно близко к точке $M$, секущая как бы достигает своего предельного положения, в котором по сути она лишь касается графика.

«Уравнение касательной» 👇

Это положение называется касательной к графику кривой $L$ в точке $M$.

Уравнение касательной через производную

Теперь узнаем, как найти уравнение касательной.

Рассмотрим некую функцию $y(x)$ и выберем на ней точку $M$ с координатами $(a; y(a))$.

Сделаем приращение к аргументу $x$ в этой точке, равное $Δx$ и рассмотрим точку $P$ на графике функции с абсциссой, равной $x=x+Δx$. Значение функции в этой точке будет равно $y(a+ Δx)$. Проведём через точки $M$ и $P$ секущую.

Как мы помним из курса математики, угловой коэффициент равен тангенсу угла прямой с осью абсцисс. Это значит, что угловой коэффициент рассматриваемой нами секущей равен приращению функции $y$ к приращению функции $x$:

$k_{секущ.}=frac{Δy}{Δx}left(1right)$.

Теперь рассмотрим приращение $Δx$ как бесконечно малую величину. В этом случае точка $P$ с координатами $(a; y(a)+ Δy)$ будет приближаться к точке $M$, стремясь к ней. Следовательно, угловой коэффициент нашей секущей, которая в данном случае является касательной, равен пределу:

$k_{кас.}=lim_{ Δx to 0}(k_{секущ.})$

Воспользуемся формулой $(1)$ для секущей:

$k_{кас.}=lim_{ Δx to 0} frac{Δy}{Δx}$

Данный предел также носит название производной функции $y=f(x)$ в точке $x$ и обозначается как $y’(x)$.

Определение 3

Геометрический смысл производной состоит в том, что при условии возможности проведения касательной в точке $x$ к графику исследуемой кривой, такой, что эта касательная не параллельна оси $OX$, значение производной является угловым коэффициентом проведённой касательной в этой точке.

Иначе данное утверждение можно записать как

$k_{кас.}(a)=f’(a)$.

То есть, при составлении уравнения касательной через производную, производная функции является угловым коэффициентом.

Заметим на всякий случай, что сама функция $y=f(x)$ и её производная $y’(x)$ — две разные функции, равные между собой в точке $x$.

Таким образом, в общем виде уравнение касательной будет иметь вид:

$y=f(x_0)+f’(x_0)(x-x_0) left(2right)$,

где $f(x_0)$ — значение функции в точке $x_0$, а $f’(x_0)$ — её производная.

Уравнение касательной для параболы

<a href=Уравнение касательной к графику параболической функции. Автор24 — интернет-биржа студенческих работ” />

Рисунок 2. Уравнение касательной к графику параболической функции. Автор24 — интернет-биржа студенческих работ

Рассмотрим получение уравнения касательной к графику функции на параболе $y=ax^2$ в точке $M$ c координатами $(x; y)$.

Придадим этой точке приращение по оси $OX$, равное $Δx$, приращение по оси $y$ тогда составит $y+Δy=a(x+ Δx)^2$. Точку с координатами $(x+ Δx; y+Δy)$ назовём $P$.

Теперь чтобы определить тангенс угла секущей $MP$с осью абсцисс, рассмотрим прямоугольный треугольник $triangle MNP$. В нём катет $MN$ равен $Δx$, а второй катет $Δy$ — это приращение ординаты, равное $Δy=a(2x cdot Δx + Δx^2)$.

Выразим используя эти данные тангенс угла $φ$.

$mathrm{tg}φ=frac{Δy}{Δx}=2ax + a cdot Δx$

Теперь для получения углового коэффициента рассмотрим это отношение при бесконечно малой величине $Δx$. Как известно, в этом случае мы имеем дело с пределом:

$mathrm{tg}φ= lim_{Δx to 0}(2ax+a cdot x)=2ax$.

Благодаря такому соотношению становится легко построить касательную к параболе (рис. 2, б).

Для этого достаточно рассмотреть треугольник $triangle MPT$, так как отрезок $TP$ будет равен:

$TP=frac{y}{mathrm{tg}α}=frac{ax^2}{2ax}=frac{x}{2}$

То есть, для того чтобы получить касательную, необходимо соединить середину отрезка $OP$ с точкой $M$.

Расположение касательной в зависимости от значения её углового коэффициента

Рассмотрим несколько различных случаев значения углового коэффициента для касательной.

Если её угловой коэффициент, то есть, тангенс, равен нулю, то касательная расположена параллельно оси $OX$, а сама прямая принимает вид $y=b$.

Если тангенс положительный, то касательная образует острый угол с осью абсцисс, что значит, что вместе с ростом $x$ растёт и $y$.

В случае если тангенс отрицательный, прямая образует тупой угол с горизонтальной осью, а это значит, что с увеличением значения икса происходит уменьшение значения игрека.

Есть ещё один случай расположения касательной — параллельно оси $OY$, в этом случае её уравнение описывается как $x=c$, где $c$ — некая константа.

Другим числом, определяющим положение касательной, является число $b$, являющееся свободным членом в уравнении прямой $y=kx+b$. Число $b$ характеризует значение функции $y(x)$ в точке её пересечения с осью ординат, иначе говоря, оно есть не что иное, как значение уравнения касательной к графику функции в точке $x=0$.

Пример 1

Составить уравнение касательной в точке $x=3$ для графика функции $y(x)=2x^2+3x-6$.

Сначала найдём значение функции в точке $x=3$:

$y=2 cdot 3^2 +3 cdot 3 – 6 = 21$

Теперь определим значение производной для исследуемой функции:

$(2x^2+3x-6)’=4x+3$

Теперь получим значение углового коэффициента, для этого подставим $x=3$ в производную:

$y’(x)=4 cdot 3 + 3 = 15$

Подставим это значение в формулу для касательной $(2)$:

$y_{кас.}=21+15 cdot (x-3)$

$y=15x-24$ — уравнение касательной получено.

Находи статьи и создавай свой список литературы по ГОСТу

Поиск по теме

Добавить комментарий